[obm-l] Re: [obm-l] Soma de raízes quadradas

2020-04-05 Por tôpico Anderson Torres
Em seg., 17 de fev. de 2020 às 12:43, Vanderlei Nemitz
 escreveu:
>
> Boa tarde!
> Existe uma fórmula fechada para a soma das raízes quadradas dos n primeiros 
> números naturais?
>

1 - Duvido.

2 - Qual a necessidade prática disso?

> Muito obrigado!
>
> --
> Esta mensagem foi verificada pelo sistema de antivírus e
> acredita-se estar livre de perigo.

-- 
Esta mensagem foi verificada pelo sistema de antiv�rus e
 acredita-se estar livre de perigo.


=
Instru��es para entrar na lista, sair da lista e usar a lista em
http://www.mat.puc-rio.br/~obmlistas/obm-l.html
=


Re: [obm-l] Soma surpreendentemente inteira

2020-03-14 Por tôpico qedtexte

Sauda,c~oes, oi Pedro,

Colocando "sum 3/(cos((24pi n)/180)-1) n=1 to 7" no WolframAlpha
o resultado  -56.

Mas no sei como fazer. Eu tentaria fazer 1=cos0 e
cos(24n)-cos0=-2sin^2(12n)

Colocando no WA

sum 3/(-2sin^2((12pi n)/180)) n=1 to 7; sum 3/(-2sin^2((pi n)/15)) n=1 to 7

ele retorna alguns clculos e volta a encontrar -56.

Agora a soma fica-3/2 sum_(n=1)^7 1/(sin^2(( n)/15))

Usando a lgebra dos nmeros complexos pode ser que saia.

Lus
--
Esta mensagem foi verificada pelo sistema de antiv�rus e
acredita-se estar livre de perigo.



Re: [obm-l] Re: [obm-l] Soma de raízes quadradas

2020-02-18 Por tôpico Maikel Andril Marcelino
http://www.mat.puc-rio.br/~nicolau/olimp/obm-l.html


Atenciosamente,

Maikel Andril Marcelino
Assistente de Aluno
Coordenadoria de Apoio Acadêmico - COAPAC/IFRN-SPP
Instituto Federal do Rio Grande do Norte
Campus São Paulo do Potengi

(84) 9-9149-8991 (Contato)
(84) 8851-3451 (WhatsApp)

De: owner-ob...@mat.puc-rio.br  em nome de Hermann 

Enviado: terça-feira, 18 de fevereiro de 2020 11:07
Para: obm-l@mat.puc-rio.br
Assunto: RES: [obm-l] Re: [obm-l] Soma de raízes quadradas

Escreve para esse email

nicolau[AT]mat.puc-rio.br ou nicolau.saldanha[AT]gmail.com

dizendo que quer sair da lista

Enviado do Email<https://go.microsoft.com/fwlink/?LinkId=550986> para Windows 10

De: Lorena Luna<mailto:lorilori20102...@hotmail.com>
Enviado:terça-feira, 18 de fevereiro de 2020 03:22
Para: obm-l@mat.puc-rio.br<mailto:obm-l@mat.puc-rio.br>
Assunto: [obm-l] Re: [obm-l] Soma de raízes quadradas

CANCELAR LISTA DE E-MAIL (Cancelar recebimento)

Em seg, 17 de fev de 2020 às 13:25, Vanderlei Nemitz 
mailto:vanderma...@gmail.com>> escreveu:
Boa tarde!
Existe uma fórmula fechada para a soma das raízes quadradas dos n primeiros 
números naturais?

Muito obrigado!

--
Esta mensagem foi verificada pelo sistema de antiv?rus e
acredita-se estar livre de perigo.

--
Esta mensagem foi verificada pelo sistema de antiv?rus e
acredita-se estar livre de perigo.


--
Esta mensagem foi verificada pelo sistema de antiv?rus e
acredita-se estar livre de perigo.

-- 
Esta mensagem foi verificada pelo sistema de antivírus e
 acredita-se estar livre de perigo.



RES: [obm-l] Re: [obm-l] Soma de raízes quadradas

2020-02-18 Por tôpico Hermann
Escreve para esse email

nicolaumat.puc-rio.br ou nicolau.saldanhagmail.com

dizendo que quer sair da lista

Enviado do Email para Windows 10

De: Lorena Luna
Enviado:terça-feira, 18 de fevereiro de 2020 03:22
Para: obm-l@mat.puc-rio.br
Assunto: [obm-l] Re: [obm-l] Soma de raízes quadradas

CANCELAR LISTA DE E-MAIL (Cancelar recebimento)

Em seg, 17 de fev de 2020 às 13:25, Vanderlei Nemitz  
escreveu:
Boa tarde!
Existe uma fórmula fechada para a soma das raízes quadradas dos n primeiros 
números naturais?

Muito obrigado!

-- 
Esta mensagem foi verificada pelo sistema de antiv�rus e 
acredita-se estar livre de perigo. 

-- 
Esta mensagem foi verificada pelo sistema de antiv�rus e 
acredita-se estar livre de perigo. 


-- 
Esta mensagem foi verificada pelo sistema de antiv�rus e
 acredita-se estar livre de perigo.



[obm-l] Re: [obm-l] Soma de raízes quadradas

2020-02-17 Por tôpico Lorena Luna
CANCELAR LISTA DE E-MAIL (Cancelar recebimento)

Em seg, 17 de fev de 2020 às 13:25, Vanderlei Nemitz 
escreveu:

> Boa tarde!
> Existe uma fórmula fechada para a soma das raízes quadradas dos n
> primeiros números naturais?
>
> Muito obrigado!
>
> --
> Esta mensagem foi verificada pelo sistema de antiv�rus e
> acredita-se estar livre de perigo.

-- 
Esta mensagem foi verificada pelo sistema de antiv�rus e
 acredita-se estar livre de perigo.



[obm-l] Re: [obm-l] Re: [obm-l] Re: [obm-l] Soma de raízes cúbicas de cossenos

2020-01-24 Por tôpico saulo nilson
NAO PRECISAVA ENCONTRAR COS5, COS 30=COS3*10, DAÍ ENCONTRA O COS10, DEPOIS
É SÓ SUBSTITUIR.

On Fri, Jan 24, 2020 at 10:23 AM Vanderlei Nemitz 
wrote:

> Como?
>
> Não entendi a ideia...
>
>
> Em sex, 24 de jan de 2020 02:37, saulo nilson 
> escreveu:
>
>> COS 15=COS 30/2
>> COS 15=COS(3*5)
>> DAÍ ENCONTRA O VALOR DE COS5 =COS10/2
>> DAÍ ENCONTRA O VALOR DE COS 10
>>
>> S= F(COS 10) QUE ENCONTRA O VALOR
>>
>> On Sun, Jan 19, 2020 at 8:41 AM Vanderlei Nemitz 
>> wrote:
>>
>>> Bom dia, pessoal!
>>>
>>> Pensei em resolver a seguinte questão associando cos 40°, cos 80° e cos
>>> 160° às raízes da equação cos(3x) = -1/2 e utilizando o arco triplo,
>>> recaindo em uma equação de grau 3. Porém, fica difícil determinar o produto
>>> de 2 em 2 das raízes cúbicas. Alguém conhece uma solução melhor?
>>> Muito obrigado!
>>>
>>> S = (cos 40°)^(1/3) + (cos 80°)^(1/3) +  (cos 160°)^(1/3)
>>>
>>> (Soma das raízes cúbicas de cos 40°, cos 80° e cos 160°)
>>>
>>> --
>>> Esta mensagem foi verificada pelo sistema de antivírus e
>>> acredita-se estar livre de perigo.
>>
>>
>> --
>> Esta mensagem foi verificada pelo sistema de antivírus e
>> acredita-se estar livre de perigo.
>
>
> --
> Esta mensagem foi verificada pelo sistema de antivírus e
> acredita-se estar livre de perigo.

-- 
Esta mensagem foi verificada pelo sistema de antiv�rus e
 acredita-se estar livre de perigo.



[obm-l] Re: [obm-l] Re: [obm-l] Soma de raízes cúbicas de cossenos

2020-01-24 Por tôpico Vanderlei Nemitz
Como?

Não entendi a ideia...


Em sex, 24 de jan de 2020 02:37, saulo nilson 
escreveu:

> COS 15=COS 30/2
> COS 15=COS(3*5)
> DAÍ ENCONTRA O VALOR DE COS5 =COS10/2
> DAÍ ENCONTRA O VALOR DE COS 10
>
> S= F(COS 10) QUE ENCONTRA O VALOR
>
> On Sun, Jan 19, 2020 at 8:41 AM Vanderlei Nemitz 
> wrote:
>
>> Bom dia, pessoal!
>>
>> Pensei em resolver a seguinte questão associando cos 40°, cos 80° e cos
>> 160° às raízes da equação cos(3x) = -1/2 e utilizando o arco triplo,
>> recaindo em uma equação de grau 3. Porém, fica difícil determinar o produto
>> de 2 em 2 das raízes cúbicas. Alguém conhece uma solução melhor?
>> Muito obrigado!
>>
>> S = (cos 40°)^(1/3) + (cos 80°)^(1/3) +  (cos 160°)^(1/3)
>>
>> (Soma das raízes cúbicas de cos 40°, cos 80° e cos 160°)
>>
>> --
>> Esta mensagem foi verificada pelo sistema de antivírus e
>> acredita-se estar livre de perigo.
>
>
> --
> Esta mensagem foi verificada pelo sistema de antivírus e
> acredita-se estar livre de perigo.

-- 
Esta mensagem foi verificada pelo sistema de antiv�rus e
 acredita-se estar livre de perigo.



[obm-l] Re: [obm-l] Soma de raízes cúbicas de cossenos

2020-01-23 Por tôpico saulo nilson
COS 15=COS 30/2
COS 15=COS(3*5)
DAÍ ENCONTRA O VALOR DE COS5 =COS10/2
DAÍ ENCONTRA O VALOR DE COS 10

S= F(COS 10) QUE ENCONTRA O VALOR

On Sun, Jan 19, 2020 at 8:41 AM Vanderlei Nemitz 
wrote:

> Bom dia, pessoal!
>
> Pensei em resolver a seguinte questão associando cos 40°, cos 80° e cos
> 160° às raízes da equação cos(3x) = -1/2 e utilizando o arco triplo,
> recaindo em uma equação de grau 3. Porém, fica difícil determinar o produto
> de 2 em 2 das raízes cúbicas. Alguém conhece uma solução melhor?
> Muito obrigado!
>
> S = (cos 40°)^(1/3) + (cos 80°)^(1/3) +  (cos 160°)^(1/3)
>
> (Soma das raízes cúbicas de cos 40°, cos 80° e cos 160°)
>
> --
> Esta mensagem foi verificada pelo sistema de antivírus e
> acredita-se estar livre de perigo.

-- 
Esta mensagem foi verificada pelo sistema de antiv�rus e
 acredita-se estar livre de perigo.



Re: Re: [obm-l] soma com cevianas que passam pelo circuncentro

2020-01-20 Por tôpico qedtexte
Sauda,c~oes, 

Essa frmula no vale para todos os tringulos obtusngulos. 
Daria para caracterizar os tringulos obtusngulos para os 
quais ela  verdadeira ? 

Abraos, 
Lus 



--
Esta mensagem foi verificada pelo sistema de antiv�rus e
acredita-se estar livre de perigo.



Re: [obm-l] Soma de Riemann

2020-01-15 Por tôpico Luiz Antonio Rodrigues
Olá, Bernardo!
Boa tarde!
Vou acessar os links que você indicou.
Muito obrigado!
Luiz

Em qua, 15 de jan de 2020 1:25 PM, Bernardo Freitas Paulo da Costa <
bernardo...@gmail.com> escreveu:

> On Tue, Jan 14, 2020 at 9:45 PM Claudio Buffara
>  wrote:
> > O artigo é esse aqui:
> >
> https://epocanegocios.globo.com/Informacao/Dilemas/noticia/2014/12/elas-precisam-de-reengenharia.html
> > É de 2014, mas ino que a situação não tenha mudado muito de lá pra cá.
>
> Há algumas tentativas de mudança.  Uma delas, o recém-criado curso de
> Engenharia Matemática da UFRJ.  Inspirado, em parte, da experiência de
> intercâmbio com a Polytechnique e a ENSTA (tanto de professores como
> de alunos), e buscando integrar a sólida formação em matemática e
> ciências básicas com o maior centro de pesquisa em engenharia da
> América Latina, a COPPE.
>
> Para mais detalhes sobre o curso, confiram
>
> https://sites.google.com/matematica.ufrj.br/aplicada/engenharia-matem%C3%A1tica
>
> http://www.im.ufrj.br/index.php/pt/noticias-e-eventos/noticias/247-saiba-mais-sobre-o-novo-curso-engenharia-matematica
>
> E, para quem quiser ler a proposta integral:
> http://www.im.ufrj.br/images/documentos/projeto_engenhariamatematica.pdf
>
> Abraços,
> --
> Bernardo Freitas Paulo da Costa
>
> --
> Esta mensagem foi verificada pelo sistema de antivírus e
>  acredita-se estar livre de perigo.
>
>
> =
> Instru�ões para entrar na lista, sair da lista e usar a lista em
> http://www.mat.puc-rio.br/~obmlistas/obm-l.html
> =
>

-- 
Esta mensagem foi verificada pelo sistema de antiv�rus e
 acredita-se estar livre de perigo.



Re: [obm-l] Soma de Riemann

2020-01-15 Por tôpico Bernardo Freitas Paulo da Costa
On Tue, Jan 14, 2020 at 9:45 PM Claudio Buffara
 wrote:
> O artigo é esse aqui:
> https://epocanegocios.globo.com/Informacao/Dilemas/noticia/2014/12/elas-precisam-de-reengenharia.html
> É de 2014, mas ino que a situação não tenha mudado muito de lá pra cá.

Há algumas tentativas de mudança.  Uma delas, o recém-criado curso de
Engenharia Matemática da UFRJ.  Inspirado, em parte, da experiência de
intercâmbio com a Polytechnique e a ENSTA (tanto de professores como
de alunos), e buscando integrar a sólida formação em matemática e
ciências básicas com o maior centro de pesquisa em engenharia da
América Latina, a COPPE.

Para mais detalhes sobre o curso, confiram
https://sites.google.com/matematica.ufrj.br/aplicada/engenharia-matem%C3%A1tica
http://www.im.ufrj.br/index.php/pt/noticias-e-eventos/noticias/247-saiba-mais-sobre-o-novo-curso-engenharia-matematica

E, para quem quiser ler a proposta integral:
http://www.im.ufrj.br/images/documentos/projeto_engenhariamatematica.pdf

Abraços,
-- 
Bernardo Freitas Paulo da Costa

-- 
Esta mensagem foi verificada pelo sistema de antiv�rus e
 acredita-se estar livre de perigo.


=
Instru��es para entrar na lista, sair da lista e usar a lista em
http://www.mat.puc-rio.br/~obmlistas/obm-l.html
=


Re: [obm-l] Soma de Riemann

2020-01-14 Por tôpico Claudio Buffara
Os livros são estes mesmo.

O artigo é esse aqui:
https://epocanegocios.globo.com/Informacao/Dilemas/noticia/2014/12/elas-precisam-de-reengenharia.html

É de 2014, mas ino que a situação não tenha mudado muito de lá pra cá.

[]s,
Claudio.


On Tue, Jan 14, 2020 at 7:09 PM Luiz Antonio Rodrigues <
rodrigue...@gmail.com> wrote:

> Olá, Claudio!
> Tudo bem?
> Muito obrigado pelas sugestões.
> Eu vi na Amazon os títulos:
>
> A Problem Book in Algebra - Krechmar
>
> Problems in Higher Algebra - Faddeev & Sominskii
>
> São esses?
> O que você disse é verdade, muitas vezes eu recorro aos softwares para
> verificar minhas respostas.
> Eu gostaria bastante de ler o artigo que você citou.
> Muito obrigado!
> Abs.
>
> Em ter, 14 de jan de 2020 5:01 PM, Claudio Buffara <
> claudio.buff...@gmail.com> escreveu:
>
>> Estas somas trigonométricas (e várias outras) são obtidas sem grandes
>> dificuldades, mas com alguma álgebra, usando números complexos.
>>
>> O melhor caminho, a meu ver, seria vc conseguir um daqueles livros russos
>> clássicos - Krechmar ou Faddev-Sominski - que contém coletâneas de
>> problemas resolvidos sobre este tema e muitos outros.
>>
>> Agora, me parece que a habilidade de computar estas somas “na mão”,
>> usando complexos e/ou alguns truques algébricos (ou até mesmo integrais)
>> tem se desvalorizado recentemente devido à existência e ampla
>> disponibilidade de softwares gratuitos tais como o Wolfram Alpha, que
>> calculam qualquer soma dessas.
>>
>> Recentemente li um artigo que toca um pouco neste tema, da necessidade de
>> modernização dos cursos de exatas. Vou procurar e postarei aqui.
>>
>> Abs
>>
>>
>> Enviado do meu iPhone
>>
>> Em 14 de jan de 2020, à(s) 12:07, Luiz Antonio Rodrigues <
>> rodrigue...@gmail.com> escreveu:
>>
>> 
>> Olá, Artur!
>> Tudo bem?
>> Agradeço sua resposta.
>> O problema diz:
>>
>> É dado o somatório de:
>>
>> sen(k*b/n)
>>
>> Onde k varia de 1 até n.
>>
>> Calcule o limite deste somatório dividido por n, quando n tende a
>> infinito.
>>
>> O problema pede que se relacione este limite com uma soma de Riemann.
>>
>> Seguindo a sugestão do Claudio, calculei o somatório dos senos em P.A.
>> Depois eu calculei o limite solicitado.
>> Cheguei n mesma resposta do Claudio, que está correta.
>> Aproveito para pedir uma indicação de material sobre este assunto, que
>> considero bastante interessante.
>> Muito obrigado!
>> Luiz
>>
>>
>> Em ter, 14 de jan de 2020 1:32 AM, Artur Costa Steiner <
>> artur.costa.stei...@gmail.com> escreveu:
>>
>>> Este somatório não é uma soma de Riemann. Seria se fosse
>>>
>>> S(n) = 1/n Soma(k = 1, n) sen(k*b/n).Â
>>>
>>> Mas é S(n) = Soma(k = 1, n) sen(k*b/n). Não se divide por n.
>>>
>>> Tem ceteza de que pelo outro processo vc chegou no seu somatório Ã
>>> expressão correspondente ao caso da soma de Riemann?
>>>
>>> Se fizermos b = pi/2, no seu somatorio temos para todo n que S(n) >
>>> sen(pi/2) = 1.Logo, se o limite com n indo para oo existir, será >= 1.Mas
>>> entrando com b = pi/2 na fórmula da soma de Riemann, obtemos 2/pi < 1.
>>>
>>> Me corrija se eu tiver cometido algum erro.
>>>
>>> Abraços
>>>
>>> Artur
>>>
>>> Em seg, 13 de jan de 2020 18:04, Luiz Antonio Rodrigues <
>>> rodrigue...@gmail.com> escreveu:
>>>
 Olá, Claudio!
 Tudo bem?
 Sim, foi esse resultado que eu achei!
 Muito obrigado pela ajuda!

 Em seg, 13 de jan de 2020 5:02 PM, Claudio Buffara <
 claudio.buff...@gmail.com> escreveu:

> É a soma de n retângulos, todos com base 1/n e o k-esimo com altura
> sen(kb/n): logo, o limite e’ a integral superior (portanto, a integral
> definida) de sen(bx) no intervalo [0,1].
>
> A antiderivada é (-1/b)*cos(bx).
>
> Logo, a integral é (1 - cos(b))/b.
>
> Enviado do meu iPhone
>
> Em 13 de jan de 2020, Ã (s) 07:04, Esdras Muniz <
> esdrasmunizm...@gmail.com> escreveu:
>
> 
> Esse limite vai ser a integral inferior de sen(x) de 0 a b. Daí,
> como Sen é integravel, esse limite vai ser Sen(b).
>
> Em dom, 12 de jan de 2020 19:19, Luiz Antonio Rodrigues <
> rodrigue...@gmail.com> escreveu:
>
>> Olá, pessoal!
>> Tudo bem?
>> Estou pensando neste problema há vários dias e não consigo
>> descobrir onde está meu erro.
>> Alguém pode me ajudar?
>>
>> O problema é o seguinte:
>>
>> É dado o somatório de:
>>
>> sen(k*b/n)
>>
>> Onde k varia de 1 até n.
>>
>> Preciso calcular o limite deste somatório dividido por n, quando n
>> tende a infinito.
>>
>> O problema pede que se relacione este limite com uma soma de Riemann.
>>
>> Eu cheguei no valor zero, que está errado.
>> O problema parece simples...
>> Agradeço desde já!
>> Luiz
>>
>>
>> --
>> Esta mensagem foi verificada pelo sistema de antivírus e
>> acredita-se estar livre de perigo.

Re: [obm-l] Soma de Riemann

2020-01-14 Por tôpico Luiz Antonio Rodrigues
Olá, Claudio!
Tudo bem?
Muito obrigado pelas sugestões.
Eu vi na Amazon os títulos:

A Problem Book in Algebra - Krechmar

Problems in Higher Algebra - Faddeev & Sominskii

São esses?
O que você disse é verdade, muitas vezes eu recorro aos softwares para
verificar minhas respostas.
Eu gostaria bastante de ler o artigo que você citou.
Muito obrigado!
Abs.

Em ter, 14 de jan de 2020 5:01 PM, Claudio Buffara <
claudio.buff...@gmail.com> escreveu:

> Estas somas trigonométricas (e várias outras) são obtidas sem grandes
> dificuldades, mas com alguma álgebra, usando números complexos.
>
> O melhor caminho, a meu ver, seria vc conseguir um daqueles livros russos
> clássicos - Krechmar ou Faddev-Sominski - que contém coletâneas de
> problemas resolvidos sobre este tema e muitos outros.
>
> Agora, me parece que a habilidade de computar estas somas “na mão”, usando
> complexos e/ou alguns truques algébricos (ou até mesmo integrais) tem se
> desvalorizado recentemente devido à existência e ampla disponibilidade de
> softwares gratuitos tais como o Wolfram Alpha, que calculam qualquer soma
> dessas.
>
> Recentemente li um artigo que toca um pouco neste tema, da necessidade de
> modernização dos cursos de exatas. Vou procurar e postarei aqui.
>
> Abs
>
>
> Enviado do meu iPhone
>
> Em 14 de jan de 2020, à(s) 12:07, Luiz Antonio Rodrigues <
> rodrigue...@gmail.com> escreveu:
>
> 
> Olá, Artur!
> Tudo bem?
> Agradeço sua resposta.
> O problema diz:
>
> É dado o somatório de:
>
> sen(k*b/n)
>
> Onde k varia de 1 até n.
>
> Calcule o limite deste somatório dividido por n, quando n tende a
> infinito.
>
> O problema pede que se relacione este limite com uma soma de Riemann.
>
> Seguindo a sugestão do Claudio, calculei o somatório dos senos em P.A.
> Depois eu calculei o limite solicitado.
> Cheguei n mesma resposta do Claudio, que está correta.
> Aproveito para pedir uma indicação de material sobre este assunto, que
> considero bastante interessante.
> Muito obrigado!
> Luiz
>
>
> Em ter, 14 de jan de 2020 1:32 AM, Artur Costa Steiner <
> artur.costa.stei...@gmail.com> escreveu:
>
>> Este somatório não é uma soma de Riemann. Seria se fosse
>>
>> S(n) = 1/n Soma(k = 1, n) sen(k*b/n).Â
>>
>> Mas é S(n) = Soma(k = 1, n) sen(k*b/n). Não se divide por n.
>>
>> Tem ceteza de que pelo outro processo vc chegou no seu somatório Ã
>> expressão correspondente ao caso da soma de Riemann?
>>
>> Se fizermos b = pi/2, no seu somatorio temos para todo n que S(n) >
>> sen(pi/2) = 1.Logo, se o limite com n indo para oo existir, será >= 1.Mas
>> entrando com b = pi/2 na fórmula da soma de Riemann, obtemos 2/pi < 1.
>>
>> Me corrija se eu tiver cometido algum erro.
>>
>> Abraços
>>
>> Artur
>>
>> Em seg, 13 de jan de 2020 18:04, Luiz Antonio Rodrigues <
>> rodrigue...@gmail.com> escreveu:
>>
>>> Olá, Claudio!
>>> Tudo bem?
>>> Sim, foi esse resultado que eu achei!
>>> Muito obrigado pela ajuda!
>>>
>>> Em seg, 13 de jan de 2020 5:02 PM, Claudio Buffara <
>>> claudio.buff...@gmail.com> escreveu:
>>>
 É a soma de n retângulos, todos com base 1/n e o k-esimo com altura
 sen(kb/n): logo, o limite e’ a integral superior (portanto, a integral
 definida) de sen(bx) no intervalo [0,1].

 A antiderivada é (-1/b)*cos(bx).

 Logo, a integral é (1 - cos(b))/b.

 Enviado do meu iPhone

 Em 13 de jan de 2020, Ã (s) 07:04, Esdras Muniz <
 esdrasmunizm...@gmail.com> escreveu:

 
 Esse limite vai ser a integral inferior de sen(x) de 0 a b. Daí,
 como Sen é integravel, esse limite vai ser Sen(b).

 Em dom, 12 de jan de 2020 19:19, Luiz Antonio Rodrigues <
 rodrigue...@gmail.com> escreveu:

> Olá, pessoal!
> Tudo bem?
> Estou pensando neste problema há vários dias e não consigo
> descobrir onde está meu erro.
> Alguém pode me ajudar?
>
> O problema é o seguinte:
>
> É dado o somatório de:
>
> sen(k*b/n)
>
> Onde k varia de 1 até n.
>
> Preciso calcular o limite deste somatório dividido por n, quando n
> tende a infinito.
>
> O problema pede que se relacione este limite com uma soma de Riemann.
>
> Eu cheguei no valor zero, que está errado.
> O problema parece simples...
> Agradeço desde já!
> Luiz
>
>
> --
> Esta mensagem foi verificada pelo sistema de antivírus e
> acredita-se estar livre de perigo.


 --
 Esta mensagem foi verificada pelo sistema de antivírus e
 acredita-se estar livre de perigo.


 --
 Esta mensagem foi verificada pelo sistema de antivírus e
 acredita-se estar livre de perigo.

>>>
>>> --
>>> Esta mensagem foi verificada pelo sistema de antivírus e
>>> acredita-se estar livre de perigo.
>>
>>
>> --
>> Esta mensagem foi verificada pelo sistema de antivírus e
>> acredita-se estar livre de perigo.
>
>
> --
> Esta mensagem 

Re: [obm-l] Soma de Riemann

2020-01-14 Por tôpico Claudio Buffara
Estas somas trigonométricas (e várias outras) são obtidas sem grandes 
dificuldades, mas com alguma álgebra, usando números complexos. 

O melhor caminho, a meu ver, seria vc conseguir um daqueles livros russos 
clássicos - Krechmar ou Faddev-Sominski - que contém coletâneas de problemas 
resolvidos sobre este tema e muitos outros.

Agora, me parece que a habilidade de computar estas somas “na mão”, usando 
complexos e/ou alguns truques algébricos (ou até mesmo integrais) tem se 
desvalorizado recentemente devido à existência e ampla disponibilidade de 
softwares gratuitos tais como o Wolfram Alpha, que calculam qualquer soma 
dessas.

Recentemente li um artigo que toca um pouco neste tema, da necessidade de 
modernização dos cursos de exatas. Vou procurar e postarei aqui.

Abs


Enviado do meu iPhone

> Em 14 de jan de 2020, à(s) 12:07, Luiz Antonio Rodrigues 
>  escreveu:
> 
> Olá, Artur!
> Tudo bem?
> Agradeço sua resposta.
> O problema diz:
> 
> É dado o somatório de:
> 
> sen(k*b/n)
> 
> Onde k varia de 1 até n.
> 
> Calcule o limite deste somatório dividido por n, quando n tende a infinito.
> 
> O problema pede que se relacione este limite com uma soma de Riemann.
> 
> Seguindo a sugestão do Claudio, calculei o somatório dos senos em P.A.
> Depois eu calculei o limite solicitado.
> Cheguei n mesma resposta do Claudio, que está correta.
> Aproveito para pedir uma indicação de material sobre este assunto, que 
> considero bastante interessante.
> Muito obrigado!
> Luiz
> 
> 
> Em ter, 14 de jan de 2020 1:32 AM, Artur Costa Steiner 
>  escreveu:
>> Este somatório não é uma soma de Riemann. Seria se fosse
>> 
>> S(n) = 1/n Soma(k = 1, n) sen(k*b/n). 
>> 
>> Mas é S(n) = Soma(k = 1, n) sen(k*b/n). Não se divide por n.
>> 
>> Tem ceteza de que pelo outro processo vc chegou no seu somatório à 
>> expressão correspondente ao caso da soma de Riemann?
>> 
>> Se fizermos b = pi/2, no seu somatorio temos para todo n que S(n) > 
>> sen(pi/2) = 1.Logo, se o limite com n indo para oo existir, será >= 1.Mas 
>> entrando com b = pi/2 na fórmula da soma de Riemann, obtemos 2/pi < 1.
>> 
>> Me corrija se eu tiver cometido algum erro.
>> 
>> Abraços
>> 
>> Artur
>> 
>> Em seg, 13 de jan de 2020 18:04, Luiz Antonio Rodrigues 
>>  escreveu:
>>> Olá, Claudio!
>>> Tudo bem?
>>> Sim, foi esse resultado que eu achei!
>>> Muito obrigado pela ajuda!
>>> 
>>> Em seg, 13 de jan de 2020 5:02 PM, Claudio Buffara 
>>>  escreveu:
 É a soma de n retângulos, todos com base 1/n e o k-esimo com altura 
 sen(kb/n): logo, o limite e’ a integral superior (portanto, a integral 
 definida) de sen(bx) no intervalo [0,1].
 
 A antiderivada é (-1/b)*cos(bx).
 
 Logo, a integral é (1 - cos(b))/b.
 
 Enviado do meu iPhone
 
> Em 13 de jan de 2020, Ã (s) 07:04, Esdras Muniz 
>  escreveu:
> 
> Esse limite vai ser a integral inferior de sen(x) de 0 a b. Daí, como 
> Sen é integravel, esse limite vai ser Sen(b).
> 
> Em dom, 12 de jan de 2020 19:19, Luiz Antonio Rodrigues 
>  escreveu:
>> Olá, pessoal!
>> Tudo bem?
>> Estou pensando neste problema há vários dias e não consigo 
>> descobrir onde está meu erro.
>> Alguém pode me ajudar?
>> 
>> O problema é o seguinte:
>> 
>> É dado o somatório de:
>> 
>> sen(k*b/n)
>> 
>> Onde k varia de 1 até n.
>> 
>> Preciso calcular o limite deste somatório dividido por n, quando n 
>> tende a infinito.
>> 
>> O problema pede que se relacione este limite com uma soma de Riemann.
>> 
>> Eu cheguei no valor zero, que está errado.
>> O problema parece simples...
>> Agradeço desde já!
>> Luiz
>> 
>> 
>> -- 
>> Esta mensagem foi verificada pelo sistema de antivírus e 
>> acredita-se estar livre de perigo.
> 
> -- 
> Esta mensagem foi verificada pelo sistema de antivírus e 
> acredita-se estar livre de perigo.
 
 -- 
 Esta mensagem foi verificada pelo sistema de antivírus e 
 acredita-se estar livre de perigo.
>>> 
>>> -- 
>>> Esta mensagem foi verificada pelo sistema de antivírus e 
>>> acredita-se estar livre de perigo.
>> 
>> -- 
>> Esta mensagem foi verificada pelo sistema de antivírus e 
>> acredita-se estar livre de perigo.
> 
> -- 
> Esta mensagem foi verificada pelo sistema de antivírus e 
> acredita-se estar livre de perigo.

-- 
Esta mensagem foi verificada pelo sistema de antiv�rus e
 acredita-se estar livre de perigo.



Re: [obm-l] Soma de Riemann

2020-01-14 Por tôpico Luiz Antonio Rodrigues
Olá, Artur!
Tudo bem?
Agradeço sua resposta.
O problema diz:

É dado o somatório de:

sen(k*b/n)

Onde k varia de 1 até n.

Calcule o limite deste somatório dividido por n, quando n tende a infinito.

O problema pede que se relacione este limite com uma soma de Riemann.

Seguindo a sugestão do Claudio, calculei o somatório dos senos em P.A.
Depois eu calculei o limite solicitado.
Cheguei n mesma resposta do Claudio, que está correta.
Aproveito para pedir uma indicação de material sobre este assunto, que
considero bastante interessante.
Muito obrigado!
Luiz


Em ter, 14 de jan de 2020 1:32 AM, Artur Costa Steiner <
artur.costa.stei...@gmail.com> escreveu:

> Este somatório não é uma soma de Riemann. Seria se fosse
>
> S(n) = 1/n Soma(k = 1, n) sen(k*b/n).
>
> Mas é S(n) = Soma(k = 1, n) sen(k*b/n). Não se divide por n.
>
> Tem ceteza de que pelo outro processo vc chegou no seu somatório à
> expressão correspondente ao caso da soma de Riemann?
>
> Se fizermos b = pi/2, no seu somatorio temos para todo n que S(n) >
> sen(pi/2) = 1.Logo, se o limite com n indo para oo existir, será >= 1.Mas
> entrando com b = pi/2 na fórmula da soma de Riemann, obtemos 2/pi < 1.
>
> Me corrija se eu tiver cometido algum erro.
>
> Abraços
>
> Artur
>
> Em seg, 13 de jan de 2020 18:04, Luiz Antonio Rodrigues <
> rodrigue...@gmail.com> escreveu:
>
>> Olá, Claudio!
>> Tudo bem?
>> Sim, foi esse resultado que eu achei!
>> Muito obrigado pela ajuda!
>>
>> Em seg, 13 de jan de 2020 5:02 PM, Claudio Buffara <
>> claudio.buff...@gmail.com> escreveu:
>>
>>> É a soma de n retângulos, todos com base 1/n e o k-esimo com altura
>>> sen(kb/n): logo, o limite e’ a integral superior (portanto, a integral
>>> definida) de sen(bx) no intervalo [0,1].
>>>
>>> A antiderivada é (-1/b)*cos(bx).
>>>
>>> Logo, a integral é (1 - cos(b))/b.
>>>
>>> Enviado do meu iPhone
>>>
>>> Em 13 de jan de 2020, à(s) 07:04, Esdras Muniz <
>>> esdrasmunizm...@gmail.com> escreveu:
>>>
>>> 
>>> Esse limite vai ser a integral inferior de sen(x) de 0 a b. Daí, como
>>> Sen é integravel, esse limite vai ser Sen(b).
>>>
>>> Em dom, 12 de jan de 2020 19:19, Luiz Antonio Rodrigues <
>>> rodrigue...@gmail.com> escreveu:
>>>
 Olá, pessoal!
 Tudo bem?
 Estou pensando neste problema há vários dias e não consigo descobrir
 onde está meu erro.
 Alguém pode me ajudar?

 O problema é o seguinte:

 É dado o somatório de:

 sen(k*b/n)

 Onde k varia de 1 até n.

 Preciso calcular o limite deste somatório dividido por n, quando n
 tende a infinito.

 O problema pede que se relacione este limite com uma soma de Riemann.

 Eu cheguei no valor zero, que está errado.
 O problema parece simples...
 Agradeço desde já!
 Luiz


 --
 Esta mensagem foi verificada pelo sistema de antivírus e
 acredita-se estar livre de perigo.
>>>
>>>
>>> --
>>> Esta mensagem foi verificada pelo sistema de antivírus e
>>> acredita-se estar livre de perigo.
>>>
>>>
>>> --
>>> Esta mensagem foi verificada pelo sistema de antivírus e
>>> acredita-se estar livre de perigo.
>>>
>>
>> --
>> Esta mensagem foi verificada pelo sistema de antivírus e
>> acredita-se estar livre de perigo.
>
>
> --
> Esta mensagem foi verificada pelo sistema de antivírus e
> acredita-se estar livre de perigo.

-- 
Esta mensagem foi verificada pelo sistema de antiv�rus e
 acredita-se estar livre de perigo.



Re: [obm-l] Soma de Riemann

2020-01-13 Por tôpico Artur Costa Steiner
Este somatório não é uma soma de Riemann. Seria se fosse

S(n) = 1/n Soma(k = 1, n) sen(k*b/n).

Mas é S(n) = Soma(k = 1, n) sen(k*b/n). Não se divide por n.

Tem ceteza de que pelo outro processo vc chegou no seu somatório à
expressão correspondente ao caso da soma de Riemann?

Se fizermos b = pi/2, no seu somatorio temos para todo n que S(n) >
sen(pi/2) = 1.Logo, se o limite com n indo para oo existir, será >= 1.Mas
entrando com b = pi/2 na fórmula da soma de Riemann, obtemos 2/pi < 1.

Me corrija se eu tiver cometido algum erro.

Abraços

Artur

Em seg, 13 de jan de 2020 18:04, Luiz Antonio Rodrigues <
rodrigue...@gmail.com> escreveu:

> Olá, Claudio!
> Tudo bem?
> Sim, foi esse resultado que eu achei!
> Muito obrigado pela ajuda!
>
> Em seg, 13 de jan de 2020 5:02 PM, Claudio Buffara <
> claudio.buff...@gmail.com> escreveu:
>
>> É a soma de n retângulos, todos com base 1/n e o k-esimo com altura
>> sen(kb/n): logo, o limite e’ a integral superior (portanto, a integral
>> definida) de sen(bx) no intervalo [0,1].
>>
>> A antiderivada é (-1/b)*cos(bx).
>>
>> Logo, a integral é (1 - cos(b))/b.
>>
>> Enviado do meu iPhone
>>
>> Em 13 de jan de 2020, à(s) 07:04, Esdras Muniz 
>> escreveu:
>>
>> 
>> Esse limite vai ser a integral inferior de sen(x) de 0 a b. Daí, como
>> Sen é integravel, esse limite vai ser Sen(b).
>>
>> Em dom, 12 de jan de 2020 19:19, Luiz Antonio Rodrigues <
>> rodrigue...@gmail.com> escreveu:
>>
>>> Olá, pessoal!
>>> Tudo bem?
>>> Estou pensando neste problema há vários dias e não consigo descobrir
>>> onde está meu erro.
>>> Alguém pode me ajudar?
>>>
>>> O problema é o seguinte:
>>>
>>> É dado o somatório de:
>>>
>>> sen(k*b/n)
>>>
>>> Onde k varia de 1 até n.
>>>
>>> Preciso calcular o limite deste somatório dividido por n, quando n
>>> tende a infinito.
>>>
>>> O problema pede que se relacione este limite com uma soma de Riemann.
>>>
>>> Eu cheguei no valor zero, que está errado.
>>> O problema parece simples...
>>> Agradeço desde já!
>>> Luiz
>>>
>>>
>>> --
>>> Esta mensagem foi verificada pelo sistema de antivírus e
>>> acredita-se estar livre de perigo.
>>
>>
>> --
>> Esta mensagem foi verificada pelo sistema de antivírus e
>> acredita-se estar livre de perigo.
>>
>>
>> --
>> Esta mensagem foi verificada pelo sistema de antivírus e
>> acredita-se estar livre de perigo.
>>
>
> --
> Esta mensagem foi verificada pelo sistema de antivírus e
> acredita-se estar livre de perigo.

-- 
Esta mensagem foi verificada pelo sistema de antiv�rus e
 acredita-se estar livre de perigo.



Re: [obm-l] Soma de Riemann

2020-01-13 Por tôpico Luiz Antonio Rodrigues
Olá, Claudio!
Tudo bem?
Sim, foi esse resultado que eu achei!
Muito obrigado pela ajuda!

Em seg, 13 de jan de 2020 5:02 PM, Claudio Buffara <
claudio.buff...@gmail.com> escreveu:

> É a soma de n retângulos, todos com base 1/n e o k-esimo com altura
> sen(kb/n): logo, o limite e’ a integral superior (portanto, a integral
> definida) de sen(bx) no intervalo [0,1].
>
> A antiderivada é (-1/b)*cos(bx).
>
> Logo, a integral é (1 - cos(b))/b.
>
> Enviado do meu iPhone
>
> Em 13 de jan de 2020, à(s) 07:04, Esdras Muniz 
> escreveu:
>
> 
> Esse limite vai ser a integral inferior de sen(x) de 0 a b. Daí, como Sen
> é integravel, esse limite vai ser Sen(b).
>
> Em dom, 12 de jan de 2020 19:19, Luiz Antonio Rodrigues <
> rodrigue...@gmail.com> escreveu:
>
>> Olá, pessoal!
>> Tudo bem?
>> Estou pensando neste problema há vários dias e não consigo descobrir
>> onde está meu erro.
>> Alguém pode me ajudar?
>>
>> O problema é o seguinte:
>>
>> É dado o somatório de:
>>
>> sen(k*b/n)
>>
>> Onde k varia de 1 até n.
>>
>> Preciso calcular o limite deste somatório dividido por n, quando n tende
>> a infinito.
>>
>> O problema pede que se relacione este limite com uma soma de Riemann.
>>
>> Eu cheguei no valor zero, que está errado.
>> O problema parece simples...
>> Agradeço desde já!
>> Luiz
>>
>>
>> --
>> Esta mensagem foi verificada pelo sistema de antivírus e
>> acredita-se estar livre de perigo.
>
>
> --
> Esta mensagem foi verificada pelo sistema de antivírus e
> acredita-se estar livre de perigo.
>
>
> --
> Esta mensagem foi verificada pelo sistema de antivírus e
> acredita-se estar livre de perigo.
>

-- 
Esta mensagem foi verificada pelo sistema de antiv�rus e
 acredita-se estar livre de perigo.



Re: [obm-l] Soma de Riemann

2020-01-13 Por tôpico Claudio Buffara
É a soma de n retângulos, todos com base 1/n e o k-esimo com altura sen(kb/n): 
logo, o limite e’ a integral superior (portanto, a integral definida) de 
sen(bx) no intervalo [0,1].

A antiderivada é (-1/b)*cos(bx).

Logo, a integral é (1 - cos(b))/b.

Enviado do meu iPhone

> Em 13 de jan de 2020, à(s) 07:04, Esdras Muniz  
> escreveu:
> 
> 
> Esse limite vai ser a integral inferior de sen(x) de 0 a b. Daí, como Sen é 
> integravel, esse limite vai ser Sen(b).
> 
> Em dom, 12 de jan de 2020 19:19, Luiz Antonio Rodrigues 
>  escreveu:
>> Olá, pessoal!
>> Tudo bem?
>> Estou pensando neste problema há vários dias e não consigo descobrir onde 
>> está meu erro.
>> Alguém pode me ajudar?
>> 
>> O problema é o seguinte:
>> 
>> É dado o somatório de:
>> 
>> sen(k*b/n)
>> 
>> Onde k varia de 1 até n.
>> 
>> Preciso calcular o limite deste somatório dividido por n, quando n tende a 
>> infinito.
>> 
>> O problema pede que se relacione este limite com uma soma de Riemann.
>> 
>> Eu cheguei no valor zero, que está errado.
>> O problema parece simples...
>> Agradeço desde já!
>> Luiz
>> 
>> 
>> -- 
>> Esta mensagem foi verificada pelo sistema de antivírus e 
>> acredita-se estar livre de perigo.
> 
> -- 
> Esta mensagem foi verificada pelo sistema de antivírus e 
> acredita-se estar livre de perigo.

-- 
Esta mensagem foi verificada pelo sistema de antiv�rus e
 acredita-se estar livre de perigo.



Re: [obm-l] Soma de Riemann

2020-01-13 Por tôpico Luiz Antonio Rodrigues
Olá, Claudio!
Olá, Esdras!
Tudo bem?
Muito obrigado pela ajuda!
Eu segui a dica do Claudio e calculei o somatório dos senos em P.A.
Depois eu calculei o limite desse somatório dividido por n.
Mas eu cheguei em

(1/b)*(1-cos(b))

O que será que houve?
Esdras,  você considerou o somatório dividido por n?


Em seg, 13 de jan de 2020 9:04 AM, Esdras Muniz 
escreveu:

> Esse limite vai ser a integral inferior de sen(x) de 0 a b. Daí, como Sen
> é integravel, esse limite vai ser Sen(b).
>
> Em dom, 12 de jan de 2020 19:19, Luiz Antonio Rodrigues <
> rodrigue...@gmail.com> escreveu:
>
>> Olá, pessoal!
>> Tudo bem?
>> Estou pensando neste problema há vários dias e não consigo descobrir onde
>> está meu erro.
>> Alguém pode me ajudar?
>>
>> O problema é o seguinte:
>>
>> É dado o somatório de:
>>
>> sen(k*b/n)
>>
>> Onde k varia de 1 até n.
>>
>> Preciso calcular o limite deste somatório dividido por n, quando n tende
>> a infinito.
>>
>> O problema pede que se relacione este limite com uma soma de Riemann.
>>
>> Eu cheguei no valor zero, que está errado.
>> O problema parece simples...
>> Agradeço desde já!
>> Luiz
>>
>>
>> --
>> Esta mensagem foi verificada pelo sistema de antivírus e
>> acredita-se estar livre de perigo.
>
>
> --
> Esta mensagem foi verificada pelo sistema de antivírus e
> acredita-se estar livre de perigo.

-- 
Esta mensagem foi verificada pelo sistema de antiv�rus e
 acredita-se estar livre de perigo.



Re: [obm-l] soma com cevianas que passam pelo circuncentro

2020-01-13 Por tôpico samuel barbosa
Olá, boa tarde.

Uma outra possibilidade:

Se r_a, r_b e r_c são as distâncias de O aos lados e h_a, h_b e h_c são as
alturas, temos

R/AO_a = (h_a-r_a)/h_a = 1 - [BOC]/[ABC].

Somando as três equações equivalentes, obtemos

R/AO_a+R/BO_b+R/CO_c = 3 - ([BOC]+[AOC]+[AOB])/[ABC] = 2.

Abraços
Samuel


Em dom., 12 de jan. de 2020 às 18:06, Anderson Torres <
torres.anderson...@gmail.com> escreveu:

> Em qua., 18 de dez. de 2019 às 20:47, Luís Lopes
>  escreveu:
> >
> > Sauda,c~oes,
> >
> > Sejam AO_a, BO_B e CO_c as cevianas que passam pelo circuncentro.
> > O_a na reta do lado  etc.
> >
> > Como provar que 1/AO_a + 1/BO_b + 1/CO_c = 2/R ?
> >
>
> Uma forma mais ou menos fácil é usando trigonometria. Calcula cada
> segmento como funçao dos ângulos e do raio do círculo, depois faz as
> contas!
>
> > Luís
> >
> >
> > --
> > Esta mensagem foi verificada pelo sistema de antivírus e
> > acredita-se estar livre de perigo.
>
> --
> Esta mensagem foi verificada pelo sistema de antivírus e
>  acredita-se estar livre de perigo.
>
>
> =
> Instru�ões para entrar na lista, sair da lista e usar a lista em
> http://www.mat.puc-rio.br/~obmlistas/obm-l.html
> =
>

-- 
Esta mensagem foi verificada pelo sistema de antiv�rus e
 acredita-se estar livre de perigo.



Re: [obm-l] Soma de Riemann

2020-01-13 Por tôpico Esdras Muniz
Esse limite vai ser a integral inferior de sen(x) de 0 a b. Daí, como Sen é
integravel, esse limite vai ser Sen(b).

Em dom, 12 de jan de 2020 19:19, Luiz Antonio Rodrigues <
rodrigue...@gmail.com> escreveu:

> Olá, pessoal!
> Tudo bem?
> Estou pensando neste problema há vários dias e não consigo descobrir onde
> está meu erro.
> Alguém pode me ajudar?
>
> O problema é o seguinte:
>
> É dado o somatório de:
>
> sen(k*b/n)
>
> Onde k varia de 1 até n.
>
> Preciso calcular o limite deste somatório dividido por n, quando n tende a
> infinito.
>
> O problema pede que se relacione este limite com uma soma de Riemann.
>
> Eu cheguei no valor zero, que está errado.
> O problema parece simples...
> Agradeço desde já!
> Luiz
>
>
> --
> Esta mensagem foi verificada pelo sistema de antivírus e
> acredita-se estar livre de perigo.

-- 
Esta mensagem foi verificada pelo sistema de antiv�rus e
 acredita-se estar livre de perigo.



Re: [obm-l] Soma de Riemann

2020-01-12 Por tôpico Claudio Buffara
Você sabe como somar os senos de arcos cujas medidas formam uma PA?
Use e^(ix) = cos(x) = i*sen(x).

On Sun, Jan 12, 2020 at 7:19 PM Luiz Antonio Rodrigues <
rodrigue...@gmail.com> wrote:

> Olá, pessoal!
> Tudo bem?
> Estou pensando neste problema há vários dias e não consigo descobrir onde
> está meu erro.
> Alguém pode me ajudar?
>
> O problema é o seguinte:
>
> É dado o somatório de:
>
> sen(k*b/n)
>
> Onde k varia de 1 até n.
>
> Preciso calcular o limite deste somatório dividido por n, quando n tende a
> infinito.
>
> O problema pede que se relacione este limite com uma soma de Riemann.
>
> Eu cheguei no valor zero, que está errado.
> O problema parece simples...
> Agradeço desde já!
> Luiz
>
>
> --
> Esta mensagem foi verificada pelo sistema de antivírus e
> acredita-se estar livre de perigo.

-- 
Esta mensagem foi verificada pelo sistema de antiv�rus e
 acredita-se estar livre de perigo.



Re: [obm-l] soma com cevianas que passam pelo circuncentro

2020-01-12 Por tôpico Anderson Torres
Em qua., 18 de dez. de 2019 às 20:47, Luís Lopes
 escreveu:
>
> Sauda,c~oes,
>
> Sejam AO_a, BO_B e CO_c as cevianas que passam pelo circuncentro.
> O_a na reta do lado  etc.
>
> Como provar que 1/AO_a + 1/BO_b + 1/CO_c = 2/R ?
>

Uma forma mais ou menos fácil é usando trigonometria. Calcula cada
segmento como funçao dos ângulos e do raio do círculo, depois faz as
contas!

> Luís
>
>
> --
> Esta mensagem foi verificada pelo sistema de antivírus e
> acredita-se estar livre de perigo.

-- 
Esta mensagem foi verificada pelo sistema de antiv�rus e
 acredita-se estar livre de perigo.


=
Instru��es para entrar na lista, sair da lista e usar a lista em
http://www.mat.puc-rio.br/~obmlistas/obm-l.html
=


Re: [obm-l] soma com cevianas que passam pelo circuncentro

2019-12-19 Por tôpico Esdras Muniz
Eu tinha feito algo parecido com essa prova 2. Usando o método k.

Em qui, 19 de dez de 2019 14:43, Luís Lopes 
escreveu:

> Sauda,c~oes,
>
> Encontrei um link com a prova:
>
> https://www.cut-the-knot.org/m/Geometry/CeviansThroughCircumcenter.shtml
>
> Esse site é muito bom.
>
> Eu conhecia a prova 3 mas não sabia que o triângulo tinha que ser
> acutângulo.
> Para triângulo retângulo vale também, por verificação direta.
>
> Aí comecei a rever a prova para triângulos obtusângulos e vi que
> havia um problema com (B-C)=90º. Acho que para triângulos obtusângulos
> a igualdade pode valer mas tem que ver para quais casos ela
> não serve. Talvez (B-C) > 90º como (115º,15º,50º) e (B-C) < 90º
> como (105º,45º,30º) satisfazem mas (B-C) = 90º como (120º,30º,30º)
> não satisfaz. Isso precisaria de outra investigação.
>
> Abraços,
> Luís
>
>
>
>
>
> --
> Esta mensagem foi verificada pelo sistema de antivírus e
> acredita-se estar livre de perigo.
>

-- 
Esta mensagem foi verificada pelo sistema de antiv�rus e
 acredita-se estar livre de perigo.



[obm-l] Re: [obm-l] Soma de frações próprias

2018-12-27 Por tôpico Anderson Torres
Em sex, 21 de dez de 2018 às 21:09, Daniel Quevedo
 escreveu:
>
> Colocando-se a fração 19/94 sob a forma 1/m + 1/n , onde m e n são inteiros 
> positivos o valor de m + n é igual a:
>

Hum...

1/m+1/n=19/94
(m+n)/(mn)=19/94

94m+94n = 19mn

19mn - 94m = 94n
m(19n-94) = 94n
19m(19n-94) = 94 * 19n
19m(19n-94) = 94 * (19n-94) + 94*94

(19m-94)(19n-94) = 94*94

Daqui já dá para prosseguir com fatorações marotas...

(19(m-5)+1)(19(n-5)+1) = 2*2*47*47

Divisores de 2*2*47*47:
1, 2, 4, 47, 94, 188, 2209, 4418, 8836

Resto 1 módulo 19:
1, 8836

Logo, m=5 e n=470

E m+n= 475, ueba!

> R: 475
> --
> Fiscal: Daniel Quevedo
>
> --
> Esta mensagem foi verificada pelo sistema de antivírus e
> acredita-se estar livre de perigo.

-- 
Esta mensagem foi verificada pelo sistema de antiv�rus e
 acredita-se estar livre de perigo.


=
Instru��es para entrar na lista, sair da lista e usar a lista em
http://www.mat.puc-rio.br/~obmlistas/obm-l.html
=


[obm-l] Re: [obm-l] Soma de frações próprias

2018-12-21 Por tôpico Pacini Bores
 

Oi Daniel, 

Faça (94-19m).(94-19n)=94^2 e  

Abraços 

Pacini 

Em 21/12/2018 21:00, Daniel Quevedo escreveu: 

> Colocando-se a fração 19/94 sob a forma 1/m + 1/n , onde m e n são inteiros 
> positivos o valor de m + n é igual a: 
> 
> R: 475 -- 
> 
> Fiscal: Daniel Quevedo 
> -- 
> Esta mensagem foi verificada pelo sistema de antivrus e 
> acredita-se estar livre de perigo.

 
-- 
Esta mensagem foi verificada pelo sistema de antiv�rus e
 acredita-se estar livre de perigo.



Re: [obm-l] Soma de binomiais

2018-11-20 Por tôpico Vanderlei Nemitz
Muito obrigado, Anderson! Vou estudar o artigo.

Em dom, 18 de nov de 2018 09:50, Anderson Torres <
torres.anderson...@gmail.com escreveu:

> Em qua, 7 de nov de 2018 às 14:38, Vanderlei Nemitz
>  escreveu:
> >
> > Boa tarde!
> > Na seguinte questão, tentei pensar no desenvolvimento de algum binômio,
> em que a parte real fosse a soma S(k), mas não consegui imaginar um.
> Fazendo alguns casos, para k de 1 a 4, conjecturei que S(k) = 2^(2k -
> 1).[2^(2k - 1) + (-1)^k].
> > Mas como posso provar que é verdadeira (se realmente for), a partir do
> zero, de preferência sem usar indução?
> >
> > Outra coisa, depois de obtida a fórmula, como obter o menor k que
> satisfaz o problema sem muitas contas? Eu testei até k igual a 14, usando
> uma calculadora.
> >
> > Obrigado!
> >
> > Seja S(k) = C(4k, 0) + C(4k, 4) + C(4k, 8) + ... + C(4k, 4k), para k =
> 1, 2, 3, 
> > O menor valor de k tal que S(k) é múltiplo de 81, é:
> > a) 7
> > b) 9
> > c) 10
> > d) 12
> > e) 14
> >
>
> Cê pode aprender sobre o Método de Multi-Secção no Problema 4 do
> artigo "Raízes da Unidade", na Eureka! 33:
>
> https://www.obm.org.br/content/uploads/2017/01/Eureka33.pdf
>
> > --
> > Esta mensagem foi verificada pelo sistema de antivírus e
> > acredita-se estar livre de perigo.
>
> --
> Esta mensagem foi verificada pelo sistema de antivírus e
>  acredita-se estar livre de perigo.
>
>
> =
> Instru�ões para entrar na lista, sair da lista e usar a lista em
> http://www.mat.puc-rio.br/~obmlistas/obm-l.html
> =
>

-- 
Esta mensagem foi verificada pelo sistema de antiv�rus e
 acredita-se estar livre de perigo.



Re: [obm-l] Soma de binomiais

2018-11-18 Por tôpico Anderson Torres
Em qua, 7 de nov de 2018 às 14:38, Vanderlei Nemitz
 escreveu:
>
> Boa tarde!
> Na seguinte questão, tentei pensar no desenvolvimento de algum binômio, em 
> que a parte real fosse a soma S(k), mas não consegui imaginar um. Fazendo 
> alguns casos, para k de 1 a 4, conjecturei que S(k) = 2^(2k - 1).[2^(2k - 1) 
> + (-1)^k].
> Mas como posso provar que é verdadeira (se realmente for), a partir do zero, 
> de preferência sem usar indução?
>
> Outra coisa, depois de obtida a fórmula, como obter o menor k que satisfaz o 
> problema sem muitas contas? Eu testei até k igual a 14, usando uma 
> calculadora.
>
> Obrigado!
>
> Seja S(k) = C(4k, 0) + C(4k, 4) + C(4k, 8) + ... + C(4k, 4k), para k = 1, 2, 
> 3, 
> O menor valor de k tal que S(k) é múltiplo de 81, é:
> a) 7
> b) 9
> c) 10
> d) 12
> e) 14
>

Cê pode aprender sobre o Método de Multi-Secção no Problema 4 do
artigo "Raízes da Unidade", na Eureka! 33:

https://www.obm.org.br/content/uploads/2017/01/Eureka33.pdf

> --
> Esta mensagem foi verificada pelo sistema de antivírus e
> acredita-se estar livre de perigo.

-- 
Esta mensagem foi verificada pelo sistema de antiv�rus e
 acredita-se estar livre de perigo.


=
Instru��es para entrar na lista, sair da lista e usar a lista em
http://www.mat.puc-rio.br/~obmlistas/obm-l.html
=


Re: [obm-l] Soma de binomiais

2018-11-08 Por tôpico Claudio Buffara
Inicialmente, sabemos que:
A = 1 + C(n,1) + C(n,2) + C(n,3) + ... = 2^n
e
B = 1 + C(n,2) + C(n,4) + ... = C(n,1) + C(n,3) + C(n,5) + ...
(basta expandir (1 + 1)^n  e  (1 - 1)^n).

Além disso:
A - B = C(n,1) + C(n,3) + C(n,5) + ... = B ==> B = A/2 = 2^(n-1)

Também temos:
(1 + i)^n = 1 + C(n,1)*i - C(n,2) - C(n,3)*i + C(n,4) + ...
(1 - i)^n = 1 - C(n,1)*i - C(n,2) + C(n,3)*i + C(n,4) + ...

De modo que:
C = ((1 + i)^n + (1 - i)^n)/2 = 1 - C(n,2) + C(n,4) - C(n,6) + ...

Logo (usando a versão "par" de B):
(B + C)/2 =  1 + C(n,4) + C(n,8) + ... = ( 2^(n-1)  +  ((1 + i)^n + (1 -
i)^n)/2 )/2

Ou seja,
1 + C(n,4) + C(n,8) + ... =  2^(n-2) + ((1 + i)^n + (1 - i)^n)/4(*)

Agora...
(1 + i)^n + (1 - i)^n = 2^(n/2)*(cis(n*pi/4) + cis(-n*pi/4)) = 2^(1 +
n/2)*cos(n*pi/4) ==>
((1 + i)^n + (1 - i)^n)/4 = 2^(n/2 - 1)*cos(n*pi/4)

Se n = 4k, então o lado direito é igual a 2^(2k-1)*cos(k*pi) =
(-1)^k*2^(2k-1)   (**)

Substituindo (**) em (*) e usando que n = 4k, teremos:
1 + C(4k,4) + C(4k,8 ) + ... + C(4k,4k) = 2^(4k-2) + (-1)^k*2^(2k-1) =
2^(2k-1) * (2^(2k-1) + (-1)^k)   (***)

***

Pra (***) ser múltiplo de 81, 2^(2k-1) + (-1)^k terá que ser múltiplo de 81.
Em particular, terá que ser múltiplo de 3.
k = 1 ==> 2^1 - 1 = 1
k = 2 ==> 2^3 + 1 = 9
k = 3 ==> 2^5 - 1 = 31
k = 4 ==> 2^7 + 1 = 129
..
Assim, parece que k = 2m + 2 (m >=0) é condição necessária.
E, de fato, k = 2m+2 ==> 2^(2k-1) + (-1)^k = 2^(4m+3) + 1 = 8*16^m + 1 ==
-1*1 + 1 == 0 (mod 3)   ("==" quer dizer "é congruente a")

Olhando mod 9, teremos 8*16^m + 1 == (-1)*(-2)^m + 1 == 0 (mod 9) sss
(-2)^m == 1 (mod 9).
Isso é verdade pra m = 0, 3, 6 e 9.
Assim, conjecturo que para m = 3p (p>=0) e, portanto, k = 6p + 2 (p >= 0),
2^(2k-1) + (-1)^k = 2^(12p+3) + 1 é múltiplo de 9.
E, de fato, olhando mod 9:  2^(12p+3) +1 == 8*4096^p + 1 == (-1)*1 + 1 == 0
(mod 9)

Testando a divisibilidade por 81:
p = 0 ==> k = 2 ==> 2^3 + 1 ==> não
p = 1 ==> k = 8 ==> 2^15 + 1 = 1024*32 + 1 == 52*32 + 1 = 1665 == 45 ==> não
p = 2 ==> k = 14 ==> 2^27 + 1 = 1024*1024*128 + 1 == 52*52*47 + 1 ==
(-29)*(-29)*(-34) + 1 == 31*(-34) + 1 = -1053 == 0 (mod 81).

Logo, o menor k é 14.

Agora, como é uma múltipla escolha, daria pra ir testando as alternativas
na expressão (***).
Acho que levaria menos tempo.

[]s,
Claudio.


On Wed, Nov 7, 2018 at 2:38 PM Vanderlei Nemitz 
wrote:

> Boa tarde!
> Na seguinte questão, tentei pensar no desenvolvimento de algum binômio, em
> que a parte real fosse a soma S(k), mas não consegui imaginar um. Fazendo
> alguns casos, para k de 1 a 4, conjecturei que S(k) = 2^(2k - 1).[2^(2k -
> 1) + (-1)^k].
> Mas como posso provar que é verdadeira (se realmente for), a partir do
> zero, de preferência sem usar indução?
>
> Outra coisa, depois de obtida a fórmula, como obter o menor k que satisfaz
> o problema sem muitas contas? Eu testei até k igual a 14, usando uma
> calculadora.
>
> Obrigado!
>
> Seja S(k) = C(4k, 0) + C(4k, 4) + C(4k, 8) + ... + C(4k, 4k), para k = 1,
> 2, 3, 
> O menor valor de k tal que S(k) é múltiplo de 81, é:
> a) 7
> b) 9
> c) 10
> d) 12
> e) 14
>
> --
> Esta mensagem foi verificada pelo sistema de antivírus e
> acredita-se estar livre de perigo.

-- 
Esta mensagem foi verificada pelo sistema de antiv�rus e
 acredita-se estar livre de perigo.



Re: [obm-l] Soma de Produtos de Termos em PA

2018-08-30 Por tôpico Claudio Buffara
Suponhamos que a sequência (a1, a2, ..., an) com n >= 3 cumpra a condição
mas não seja PA.
Seja p o menor índice tal que:
(a1, ..., a(p-1)) é PA (digamos, de razão r) mas (a1, ..., a(p-1), ap) não
é PA.
Isso significa que ap - a(p-1) <> r  (&&&)

Como (a1, ..., a(p-1)) é PA, vale:
1/(a1*a2) + ... + 1/(a(p-2)*a(p-1)) = (p-2)/(a1*a(p-1))(*)

Além disso, por hipótese:
1/(a1*a2) + ... + 1/(a(p-1)*ap) = (p-1)/(a1*ap)(**)

Subtraindo (*) de (**), obtemos:
1/(a(p-1)*ap) = (1/a1)*((p-1)/ap - (p-2)/a(p-1))  ==>
a1/(a(p-1)*ap) = ((p-1)*a(p-1) - (p-2)*ap)/(ap*a(p-1)) ==>
a1 = a(p-1) + (p-2)*a(p-1) - (p-2)*ap ==>
a(p-1) = a1 + (p-2)*(ap - a(p-1))(***)

Mas, como (a1, ..., a(p-1)) é uma PA, vale a(p-1) = a1 + (p-2)*r  (),
onde r = razão da PA.
Comparando (***) e (), obtemos que ap - a(p-1) = r ==> contradição a
(&&&).

Logo, se uma sequência cumpre a condição, ela é PA.

[]s,
Claudio.


On Thu, Aug 30, 2018 at 3:21 PM Luís Lopes  wrote:

> Sauda,c~oes, oi Claudio,
>
> Seja S_{k-1} = (n-1)/(a1*an) = \frac{n-1}{a_1a_n}.
>
> Para provar a recíproca escrevi
>
> S_k = S_{k-1} + \frac{1}{a_n a_{n+1}} = \frac{n}{a_1a_{n+1}}
>
> e cheguei a
>
> n(a_{n+1} - a_n)=a_{n+1} - a_1 (*).
>
> Fazendo a) n=2 e b) n=3 em (*) tem-se
>
> a) a_3 + a_1 = 2a_2
>
> b) a_4 + a_2 = 2a_3
>
> Mas não consegui provar que a_{k+1} + a_{k-1} = 2a_k .
>
> Usando (*) ou de outra maneira, como provar a recíproca ?
>
> []s
> Luís
>
>
>
> --
> Esta mensagem foi verificada pelo sistema de antivírus e
> acredita-se estar livre de perigo.
>

-- 
Esta mensagem foi verificada pelo sistema de antiv�rus e
 acredita-se estar livre de perigo.



Re: [obm-l] Soma de Produtos de Termos em PA

2018-08-29 Por tôpico Kevin Felipe Kuhl Oliveira
Obrigado a todos!

Eu vou verificar se houve um erro de escrita. Provavelmente existe uma 
inconsistência mesmo. Legal essa propriedade da soma dos inversos dos produtos.

Um abraço

Kevin Kühl
On 29 Aug 2018 11:50 -0300, Claudio Buffara , wrote:
> A soma que você quer talvez seja a dos inversos dos produtos de termos 
> consecutivos.
> Numa PA a1, a2, ..., an, vale:
> 1/(a1*a2) + 1/(a2*a3) + ... + 1/(a(n-1)*an) = (n-1)/(a1*an).
>
> E vale também a recíproca: se uma sequência (a1, a2, a3, ...) é tal que para 
> todo n>=3 vale a igualdade acima, então a sequência é uma PA.
>
> []s,
> Claudio.
>
>
>
> > On Wed, Aug 29, 2018 at 9:28 AM Kevin Felipe Kuhl Oliveira 
> >  wrote:
> > > Bom dia, vocês já viram o seguinte problema?
> > >
> > > Sejam a1, a2, a3, ..., an termos consecutivos, não nulos, de uma PA, 
> > > nessa ordem. Mostre que
> > >
> > > (a1*a2) + (a2*a3) + ... + (a(n-1)*an) = (n-1)(a1*an)
> > >
> > > Na minha resposta aparece um termo com r^2 ao final, então não consigo 
> > > provar. Se alguém puder ajudar, agradeço.
> > >
> > > Um abraço
> > >
> > > Kevin Kühl
> > >
> > > --
> > > Esta mensagem foi verificada pelo sistema de antivírus e
> > > acredita-se estar livre de perigo.
>
> --
> Esta mensagem foi verificada pelo sistema de antiv�rus e
> acredita-se estar livre de perigo.

-- 
Esta mensagem foi verificada pelo sistema de antiv�rus e
 acredita-se estar livre de perigo.



Re: [obm-l] Soma de Produtos de Termos em PA

2018-08-29 Por tôpico Claudio Buffara
A soma que você quer talvez seja a dos inversos dos produtos de termos
consecutivos.
Numa PA a1, a2, ..., an, vale:
1/(a1*a2) + 1/(a2*a3) + ... + 1/(a(n-1)*an) = (n-1)/(a1*an).

E vale também a recíproca: se uma sequência (a1, a2, a3, ...) é tal que
para todo n>=3 vale a igualdade acima, então a sequência é uma PA.

[]s,
Claudio.



On Wed, Aug 29, 2018 at 9:28 AM Kevin Felipe Kuhl Oliveira <
kevin_k...@usp.br> wrote:

> Bom dia, vocês já viram o seguinte problema?
>
> Sejam a1, a2, a3, ..., an termos consecutivos, não nulos, de uma PA, nessa
> ordem. Mostre que
>
> (a1*a2) + (a2*a3) + ... + (a(n-1)*an) = (n-1)(a1*an)
>
> Na minha resposta aparece um termo com r^2 ao final, então não consigo
> provar. Se alguém puder ajudar, agradeço.
>
> Um abraço
>
> Kevin Kühl
>
> --
> Esta mensagem foi verificada pelo sistema de antivírus e
> acredita-se estar livre de perigo.
>

-- 
Esta mensagem foi verificada pelo sistema de antiv�rus e
 acredita-se estar livre de perigo.



Re: [obm-l] Soma de Produtos de Termos em PA

2018-08-29 Por tôpico Artur Steiner
Isso não é verdade. Se n  3,
a1 = 1, a2= 2, a3 = 3 então

a1 a2 + a2 a3 = 8
(n - 1) a1 an = 6

Não seria 2/(a1 a2) ... + 1/(a(n -1) an) = (n -1)/(a1 an)? Isso é verdade.

Artur Costa Steiner


Em qua, 29 de ago de 2018 09:28, Kevin Felipe Kuhl Oliveira <
kevin_k...@usp.br> escreveu:

> Bom dia, vocês já viram o seguinte problema?
>
> Sejam a1, a2, a3, ..., an termos consecutivos, não nulos, de uma PA, nessa
> ordem. Mostre que
>
> (a1*a2) + (a2*a3) + ... + (a(n-1)*an) = (n-1)(a1*an)
>
> Na minha resposta aparece um termo com r^2 ao final, então não consigo
> provar. Se alguém puder ajudar, agradeço.
>
> Um abraço
>
> Kevin Kühl
>
> --
> Esta mensagem foi verificada pelo sistema de antivírus e
> acredita-se estar livre de perigo.
>

-- 
Esta mensagem foi verificada pelo sistema de antiv�rus e
 acredita-se estar livre de perigo.



Re: [obm-l] Soma de Produtos de Termos em PA

2018-08-29 Por tôpico Claudio Buffara
Tá certo isso?
Tome a PA (1,2,3,4)  a1 = 1, an = n = 4
soma = 1*2 + 2*3 + 3*4 = 2 + 6 + 12 = 20.
Mas (n-1)*a1*an = 3*1*4 = 12.



On Wed, Aug 29, 2018 at 9:38 AM Claudio Buffara 
wrote:

> an = a1 + (n-1)r ==> r = (an - a1)/(n-1) ==> r^2 = (an - a1)^2/(n-1)^2.
> Use esta expressão pra r^2. Com alguma álgebra você deve chegar lá.
>
>
> On Wed, Aug 29, 2018 at 9:28 AM Kevin Felipe Kuhl Oliveira <
> kevin_k...@usp.br> wrote:
>
>> Bom dia, vocês já viram o seguinte problema?
>>
>> Sejam a1, a2, a3, ..., an termos consecutivos, não nulos, de uma PA,
>> nessa ordem. Mostre que
>>
>> (a1*a2) + (a2*a3) + ... + (a(n-1)*an) = (n-1)(a1*an)
>>
>> Na minha resposta aparece um termo com r^2 ao final, então não consigo
>> provar. Se alguém puder ajudar, agradeço.
>>
>> Um abraço
>>
>> Kevin Kühl
>>
>> --
>> Esta mensagem foi verificada pelo sistema de antivírus e
>> acredita-se estar livre de perigo.
>>
>

-- 
Esta mensagem foi verificada pelo sistema de antiv�rus e
 acredita-se estar livre de perigo.



Re: [obm-l] Soma de Produtos de Termos em PA

2018-08-29 Por tôpico Claudio Buffara
an = a1 + (n-1)r ==> r = (an - a1)/(n-1) ==> r^2 = (an - a1)^2/(n-1)^2.
Use esta expressão pra r^2. Com alguma álgebra você deve chegar lá.


On Wed, Aug 29, 2018 at 9:28 AM Kevin Felipe Kuhl Oliveira <
kevin_k...@usp.br> wrote:

> Bom dia, vocês já viram o seguinte problema?
>
> Sejam a1, a2, a3, ..., an termos consecutivos, não nulos, de uma PA, nessa
> ordem. Mostre que
>
> (a1*a2) + (a2*a3) + ... + (a(n-1)*an) = (n-1)(a1*an)
>
> Na minha resposta aparece um termo com r^2 ao final, então não consigo
> provar. Se alguém puder ajudar, agradeço.
>
> Um abraço
>
> Kevin Kühl
>
> --
> Esta mensagem foi verificada pelo sistema de antivírus e
> acredita-se estar livre de perigo.
>

-- 
Esta mensagem foi verificada pelo sistema de antiv�rus e
 acredita-se estar livre de perigo.



Re: [obm-l] Soma (k = 1, n) 1/P'(r_k) = 0

2018-04-16 Por tôpico gugu
   Dá para fazer com interpolação de Lagrange: o único polinômio de  
grau <=n-1 que vale 1 em r_i para 1<=i<=n (que obviamente é o  
polinômio constante igual a 1) é dado por
soma_(1<=i<=n)Produto_(j<>i)((x-r_j)/(r_i-r_j)). Aí é só olhar para o  
coeficiente de x^[n-1} (que é 0).

   Abraços,
 Gugu

Quoting Claudio Buffara :


Resumo da ópera: ainda não temos uma demonstração elementar disso.

Mas não deixa de ser interessante tentar dar uma interpretação geométrica
da expressão para polinômios de grau baixo que tenham todas as raízes reais
e distintas.

Grau 2 é meio evidente:  as retas tangentes à parábola nas raízes têm
inclinações de mesma magnitude e sinais opostos.

Grau 3 é mais interessante...
De cara, dá margem ao problema: "Uma função polinomial de grau 3 tem raízes
(ou será que é melhor chamar de "zeros" - funções têm zeros, quem tem
raízes são equações...) reais e distintas: a, b e c. Se as retas tangentes
ao gráfico da função nos pontos (a,0) e (b,0) têm inclinações m e n,
respectivamente, determine a inclinação da reta tangente ao gráfico no
ponto (c,0).", cuja tentativa mais óbvia de solução vai descambar num monte
de contas (desnecessárias) envolvendo a, b e c, quando a resposta é
-mn/(m+n).

[]s,
Claudio.

2018-04-16 14:10 GMT-03:00 Douglas Oliveira de Lima <
profdouglaso.del...@gmail.com>:


Entao a questao é até que ponto ela é verdadeira , pois funciona para
casos elementares.

Douglas Oliveira

Em dom, 15 de abr de 2018 22:29, Bernardo Freitas Paulo da Costa <
bernardo...@gmail.com> escreveu:


2018-04-15 13:09 GMT-03:00 Douglas Oliveira de Lima
:
> Usa o polinomio de Lagrange , nao é nada obvia mesmo.

Como usa Lagrange, a fórmula segue para k = 0, 1, ... n-1
(interpolando em n pontos, vamos até grau n-1).  E é, de fato, falso
para k = n, use P(x) = (x-1)(x+1).

Além disso, mesmo para k = n-1, a demonstração por complexa não se
aplica mais (o grau dá errado...), e o mesmo polinômio serve para
mostrar que a soma não dá mais zero.

Abraços,
--
Bernardo Freitas Paulo da Costa

--
Esta mensagem foi verificada pelo sistema de antivírus e
 acredita-se estar livre de perigo.


=
Instru�ões para entrar na lista, sair da lista e usar a lista em
http://www.mat.puc-rio.br/~obmlistas/obm-l.html
=



--
Esta mensagem foi verificada pelo sistema de antivírus e
acredita-se estar livre de perigo.



--
Esta mensagem foi verificada pelo sistema de antiv?rus e
 acredita-se estar livre de perigo.





--
Esta mensagem foi verificada pelo sistema de antiv�rus e
acredita-se estar livre de perigo.

=
Instru��es para entrar na lista, sair da lista e usar a lista em
http://www.mat.puc-rio.br/~obmlistas/obm-l.html
=


Re: [obm-l] Soma (k = 1, n) 1/P'(r_k) = 0

2018-04-16 Por tôpico Bernardo Freitas Paulo da Costa
2018-04-16 20:54 GMT-03:00 Claudio Buffara :
> Resumo da ópera: ainda não temos uma demonstração elementar disso.
>
> Mas não deixa de ser interessante tentar dar uma interpretação geométrica da
> expressão para polinômios de grau baixo que tenham todas as raízes reais e
> distintas.
>
> Grau 2 é meio evidente:  as retas tangentes à parábola nas raízes têm
> inclinações de mesma magnitude e sinais opostos.

A média harmônica das inclinações é zero, o que mesmo algébrico, não
deixa de ser interessante.  E talvez "baste" achar uma visão
geométrica da média harmônica neste contexto. Outra forma de dar a
mesma equação é que "a última inclinação" é (o oposto) da média
harmônica das outras.  Como interpretar isso, tenho menos ideia
ainda...


> Grau 3 é mais interessante...
> De cara, dá margem ao problema: "Uma função polinomial de grau 3 tem raízes
> (ou será que é melhor chamar de "zeros" - funções têm zeros, quem tem raízes
> são equações...) reais e distintas: a, b e c. Se as retas tangentes ao
> gráfico da função nos pontos (a,0) e (b,0) têm inclinações m e n,
> respectivamente, determine a inclinação da reta tangente ao gráfico no ponto
> (c,0).", cuja tentativa mais óbvia de solução vai descambar num monte de
> contas (desnecessárias) envolvendo a, b e c, quando a resposta é -mn/(m+n).

Também é verdade (graças ao Douglas e o Polinômio Interpolador de
Lagrange) que a/m + b/n - c(m+n)/mn = 0, ou seja, an + bm = c(m+n), o
que dá uma relação entre m e n em função das raízes.  Observe que isso
está "certo" (de novo do ponto de vista algébrico), pois o polinômio
tem 4 coeficientes, e além das 3 raízes, precisamos de um fator
multiplicativo, que pode ser dado de várias formas: tipicamente, é o
coeficiente de mais alto grau ou o valor em um ponto particular que
não seja zero, mas agora acabamos de ver que poderia até ser a
inclinação de uma tangente a uma raiz simples! (a, b, c, m determinam
tudo!)

Aliás, isso sugere que talvez haja uma demonstração puramente
algébrica para tudo isso, contando dimensões.  O número de
coeficientes do polinômio é (n+1).  Por outro lado, dados n valores
x_i (para as raízes), e n valores m_i (para as derivadas nestes
pontos), sabemos que há n-1 equações G(X_i, M_i) = 0 para "acertar a
dimensão".  Claro que as equações devem ser simétricas nos X_i e
M_i... mas isso ainda não basta para mostrar a forma especial \sum
X_i^k / M_i = 0...  Alguém tem uma ideia?  Por exemplo, já pode ser um
bom passo mostrar que as equações são homogêneas em X e M.

Abraços,
-- 
Bernardo Freitas Paulo da Costa

-- 
Esta mensagem foi verificada pelo sistema de antiv�rus e
 acredita-se estar livre de perigo.


=
Instru��es para entrar na lista, sair da lista e usar a lista em
http://www.mat.puc-rio.br/~obmlistas/obm-l.html
=


Re: [obm-l] Soma (k = 1, n) 1/P'(r_k) = 0

2018-04-16 Por tôpico Claudio Buffara
Resumo da ópera: ainda não temos uma demonstração elementar disso.

Mas não deixa de ser interessante tentar dar uma interpretação geométrica
da expressão para polinômios de grau baixo que tenham todas as raízes reais
e distintas.

Grau 2 é meio evidente:  as retas tangentes à parábola nas raízes têm
inclinações de mesma magnitude e sinais opostos.

Grau 3 é mais interessante...
De cara, dá margem ao problema: "Uma função polinomial de grau 3 tem raízes
(ou será que é melhor chamar de "zeros" - funções têm zeros, quem tem
raízes são equações...) reais e distintas: a, b e c. Se as retas tangentes
ao gráfico da função nos pontos (a,0) e (b,0) têm inclinações m e n,
respectivamente, determine a inclinação da reta tangente ao gráfico no
ponto (c,0).", cuja tentativa mais óbvia de solução vai descambar num monte
de contas (desnecessárias) envolvendo a, b e c, quando a resposta é
-mn/(m+n).

[]s,
Claudio.

2018-04-16 14:10 GMT-03:00 Douglas Oliveira de Lima <
profdouglaso.del...@gmail.com>:

> Entao a questao é até que ponto ela é verdadeira , pois funciona para
> casos elementares.
>
> Douglas Oliveira
>
> Em dom, 15 de abr de 2018 22:29, Bernardo Freitas Paulo da Costa <
> bernardo...@gmail.com> escreveu:
>
>> 2018-04-15 13:09 GMT-03:00 Douglas Oliveira de Lima
>> :
>> > Usa o polinomio de Lagrange , nao é nada obvia mesmo.
>>
>> Como usa Lagrange, a fórmula segue para k = 0, 1, ... n-1
>> (interpolando em n pontos, vamos até grau n-1).  E é, de fato, falso
>> para k = n, use P(x) = (x-1)(x+1).
>>
>> Além disso, mesmo para k = n-1, a demonstração por complexa não se
>> aplica mais (o grau dá errado...), e o mesmo polinômio serve para
>> mostrar que a soma não dá mais zero.
>>
>> Abraços,
>> --
>> Bernardo Freitas Paulo da Costa
>>
>> --
>> Esta mensagem foi verificada pelo sistema de antivírus e
>>  acredita-se estar livre de perigo.
>>
>>
>> =
>> Instru�ões para entrar na lista, sair da lista e usar a lista em
>> http://www.mat.puc-rio.br/~obmlistas/obm-l.html
>> =
>>
>
> --
> Esta mensagem foi verificada pelo sistema de antivírus e
> acredita-se estar livre de perigo.
>

-- 
Esta mensagem foi verificada pelo sistema de antiv�rus e
 acredita-se estar livre de perigo.



Re: [obm-l] Soma (k = 1, n) 1/P'(r_k) = 0

2018-04-16 Por tôpico Artur Steiner
É verdadeira para todo polinômio de grau  n >= 2 que tenha n raízes
simples.

Artur Costa Steiner

Em Seg, 16 de abr de 2018 14:18, Douglas Oliveira de Lima <
profdouglaso.del...@gmail.com> escreveu:

> Entao a questao é até que ponto ela é verdadeira , pois funciona para
> casos elementares.
>
> Douglas Oliveira
>
> Em dom, 15 de abr de 2018 22:29, Bernardo Freitas Paulo da Costa <
> bernardo...@gmail.com> escreveu:
>
>> 2018-04-15 13:09 GMT-03:00 Douglas Oliveira de Lima
>> :
>> > Usa o polinomio de Lagrange , nao é nada obvia mesmo.
>>
>> Como usa Lagrange, a fórmula segue para k = 0, 1, ... n-1
>> (interpolando em n pontos, vamos até grau n-1).  E é, de fato, falso
>> para k = n, use P(x) = (x-1)(x+1).
>>
>> Além disso, mesmo para k = n-1, a demonstração por complexa não se
>> aplica mais (o grau dá errado...), e o mesmo polinômio serve para
>> mostrar que a soma não dá mais zero.
>>
>> Abraços,
>> --
>> Bernardo Freitas Paulo da Costa
>>
>> --
>> Esta mensagem foi verificada pelo sistema de antivírus e
>>  acredita-se estar livre de perigo.
>>
>>
>> =
>> Instru�ões para entrar na lista, sair da lista e usar a lista em
>> http://www.mat.puc-rio.br/~obmlistas/obm-l.html
>> =
>>
>
> --
> Esta mensagem foi verificada pelo sistema de antivírus e
> acredita-se estar livre de perigo.

-- 
Esta mensagem foi verificada pelo sistema de antiv�rus e
 acredita-se estar livre de perigo.



Re: [obm-l] Soma (k = 1, n) 1/P'(r_k) = 0

2018-04-16 Por tôpico Douglas Oliveira de Lima
Entao a questao é até que ponto ela é verdadeira , pois funciona para casos
elementares.

Douglas Oliveira

Em dom, 15 de abr de 2018 22:29, Bernardo Freitas Paulo da Costa <
bernardo...@gmail.com> escreveu:

> 2018-04-15 13:09 GMT-03:00 Douglas Oliveira de Lima
> :
> > Usa o polinomio de Lagrange , nao é nada obvia mesmo.
>
> Como usa Lagrange, a fórmula segue para k = 0, 1, ... n-1
> (interpolando em n pontos, vamos até grau n-1).  E é, de fato, falso
> para k = n, use P(x) = (x-1)(x+1).
>
> Além disso, mesmo para k = n-1, a demonstração por complexa não se
> aplica mais (o grau dá errado...), e o mesmo polinômio serve para
> mostrar que a soma não dá mais zero.
>
> Abraços,
> --
> Bernardo Freitas Paulo da Costa
>
> --
> Esta mensagem foi verificada pelo sistema de antivírus e
>  acredita-se estar livre de perigo.
>
>
> =
> Instru�ões para entrar na lista, sair da lista e usar a lista em
> http://www.mat.puc-rio.br/~obmlistas/obm-l.html
> =
>

-- 
Esta mensagem foi verificada pelo sistema de antiv�rus e
 acredita-se estar livre de perigo.



Re: Re: [obm-l] Soma (k = 1, n) 1/P'(r_k) = 0

2018-04-16 Por tôpico qedtexte

Sauda,c~oes,

Mandei a mensagem abaixo dessa na 6a.feira mas acho que no chegou.

Terminei a dita mensagem com a pergunta


Como concluir (seria possvel ?) a partir de (*)
que \sum_{k=1}^{n} \frac{1}{Q'(a_k)} = 0 ?

Na verdade o Gugu provara (*) para o caso real, ou seja, Q(x) e no
Q(z). Talvez seja possvel provar (*) para Q(z) com alguma 
adaptao.
Aqui vou provar a soma pedida para os polinmios com os coeficientes
reais.

Seja



1/Q(x) = \sum_{k=1}^{n} \frac{1}{Q'(a_k)}\frac{1}{x-a_k} (*)


obtida a partir de


\frac{P(x)}{Q(x)} = \sum_{k=1}^{n} \frac{P(a_k)}{Q'(a_k)}\frac{1}{x-a_k}

Igualando o denominador de todas as parcelas do somatrio (*) 
obtm-se
um polinmio no numerador cujo termo lder  x^{n-1} e seu 
coeficiente
vale \sum_{k=1}^{n} \frac{1}{Q'(a_k)}. Ento para n=2 esta soma tem 
que
ser igual a zero pois por (*) o polinmio obtido s possui o termo 
independente
1.

Abs,
Lus



 mensagem enviada na ltima 6a.feira
Sauda,c~oes,

H algum (bastante) tempo atrs o Gugu (seme permitem)
mandou para a lista a prova do seguinte resultado:


Sejam P(z) e Q(z) dois polinmios, de graus m e n, respectivamente,
e mn. Se todas as n razes a_k de Q(z) so razes 
simples,
ento a decomposio em fraes parciais de 
P(z)/Q(z)
pode ser expressa da seguinte maneira (\frac{A}{B}=A/B) :

\frac{P(z)}{Q(z)} = \sum_{k=1}^{n} \frac{P(a_k)}{Q'(a_k)}\frac{1}{z-a_k}
onde Q'(z)=\frac{d}{dz}Q(z).


Ento se P(z) = 1 para todo z,


1/Q(z) = \sum_{k=1}^{n} \frac{1}{Q'(a_k)}\frac{1}{z-a_k} (*)

Como concluir (seria possvel ?) a partir de (*)
que \sum_{k=1}^{n} \frac{1}{Q'(a_k)} = 0 ?

Abs,
Lus

--
Esta mensagem foi verificada pelo sistema de antiv�rus e
acredita-se estar livre de perigo.



Re: [obm-l] Soma (k = 1, n) 1/P'(r_k) = 0

2018-04-15 Por tôpico Bernardo Freitas Paulo da Costa
2018-04-15 13:09 GMT-03:00 Douglas Oliveira de Lima
:
> Usa o polinomio de Lagrange , nao é nada obvia mesmo.

Como usa Lagrange, a fórmula segue para k = 0, 1, ... n-1
(interpolando em n pontos, vamos até grau n-1).  E é, de fato, falso
para k = n, use P(x) = (x-1)(x+1).

Além disso, mesmo para k = n-1, a demonstração por complexa não se
aplica mais (o grau dá errado...), e o mesmo polinômio serve para
mostrar que a soma não dá mais zero.

Abraços,
-- 
Bernardo Freitas Paulo da Costa

-- 
Esta mensagem foi verificada pelo sistema de antiv�rus e
 acredita-se estar livre de perigo.


=
Instru��es para entrar na lista, sair da lista e usar a lista em
http://www.mat.puc-rio.br/~obmlistas/obm-l.html
=


Re: [obm-l] Soma (k = 1, n) 1/P'(r_k) = 0

2018-04-15 Por tôpico Douglas Oliveira de Lima
Usa o polinomio de Lagrange , nao é nada obvia mesmo.

Douglas Oliveira.

Em sex, 13 de abr de 2018 13:41, Claudio Buffara 
escreveu:

> Essa identidade:
>  x^k=soma (i=1,...,n)(x_i)^k.P(x)/(x-x_i).P'(x_i)
> não me parece nada óbvia.
>
> []s,
> Claudio.
>
>
> 2018-04-13 5:56 GMT-03:00 Douglas Oliveira de Lima <
> profdouglaso.del...@gmail.com>:
>
>> Entao, sendo x^k=soma (i=1,...,n)(x_i)^k.P(x)/(x-x_i).P'(x_i) , é só
>> igualar os coeficientes de x^(n-1) e pronto, a identidade se torna ate mais
>> genérica
>>
>> Soma (i= 1, n) (x_i)^k/P'(x_i) = 0
>>
>> Obs: x_i sao raizes.
>>
>> Abraco
>>
>> Douglas Oliveira.
>>
>>
>>
>>
>> Em 8 de abr de 2018 20:50, "Artur Steiner" 
>> escreveu:
>>
>> Seja P um polinômio complexo, de grau n >= 2, que tenha n raízes simples
>> r_1, ... r_n. Mostre que Soma (k = 1, n) 1/P'(r_k) = 0.
>>
>> Para quem conhece um pouco de análise complexa, isto é corolário de um
>> resultado geral. Mas parece que pode ser provado sem análise complexa.
>>
>> Artur Costa Steiner
>>
>> --
>> Esta mensagem foi verificada pelo sistema de antivírus e
>> acredita-se estar livre de perigo.
>>
>>
>>
>> --
>> Esta mensagem foi verificada pelo sistema de antivírus e
>> acredita-se estar livre de perigo.
>>
>
>
> --
> Esta mensagem foi verificada pelo sistema de antivírus e
> acredita-se estar livre de perigo.

-- 
Esta mensagem foi verificada pelo sistema de antiv�rus e
 acredita-se estar livre de perigo.



Re: [obm-l] Soma (k = 1, n) 1/P'(r_k) = 0

2018-04-13 Por tôpico Claudio Buffara
Essa identidade:
 x^k=soma (i=1,...,n)(x_i)^k.P(x)/(x-x_i).P'(x_i)
não me parece nada óbvia.

[]s,
Claudio.


2018-04-13 5:56 GMT-03:00 Douglas Oliveira de Lima <
profdouglaso.del...@gmail.com>:

> Entao, sendo x^k=soma (i=1,...,n)(x_i)^k.P(x)/(x-x_i).P'(x_i) , é só
> igualar os coeficientes de x^(n-1) e pronto, a identidade se torna ate mais
> genérica
>
> Soma (i= 1, n) (x_i)^k/P'(x_i) = 0
>
> Obs: x_i sao raizes.
>
> Abraco
>
> Douglas Oliveira.
>
>
>
>
> Em 8 de abr de 2018 20:50, "Artur Steiner" 
> escreveu:
>
> Seja P um polinômio complexo, de grau n >= 2, que tenha n raízes simples
> r_1, ... r_n. Mostre que Soma (k = 1, n) 1/P'(r_k) = 0.
>
> Para quem conhece um pouco de análise complexa, isto é corolário de um
> resultado geral. Mas parece que pode ser provado sem análise complexa.
>
> Artur Costa Steiner
>
> --
> Esta mensagem foi verificada pelo sistema de antivírus e
> acredita-se estar livre de perigo.
>
>
>
> --
> Esta mensagem foi verificada pelo sistema de antivírus e
> acredita-se estar livre de perigo.
>

-- 
Esta mensagem foi verificada pelo sistema de antiv�rus e
 acredita-se estar livre de perigo.



Re: [obm-l] Soma (k = 1, n) 1/P'(r_k) = 0

2018-04-13 Por tôpico Artur Steiner
A prova por análise complexa baseia-se no fato de que, se P e Q  são
polinômios com grau(P) >= grau(Q) + 2 e f =Q/P, definida para P(z) <> 0,
então,

Soma (z em Z) Res(f, z) = 0  (*)

onde Z é o conjunto dos zeros de P e Res(f, z) é o resíduo de f em z, que é
pólo de f.

A prova disso baseia-se no teorema dos resíduos e no fato de que lim r -->
oo Integral (sobre Cr) f(z) dz = 0, sendo Cr a periferia do disco de centro
na origem e raio r.

Particularizando-se para o caso em que Q(z) = 1 para todo z,  f = 1/P e (*)
se reduz a

Soma (z em Z) Res(1/P, z) = 0 (**)

Se z é zero simples de P, então Res(1/P, z) = 1/P'(z), pois z é pólo
simples de 1/P (observe que, neste caso,  P'(z) <> 0). Supondo-se agora que
P tem grau n >= 2 e tem n zeros simples r_1,  r_n, (**) implica que

Soma (k = 1, n) 1/P'(r_k) = 0

conforme afirmado.

A chave da prova é que

lim r --> oo Integral (sobre Cr) f(z) dz = 0,

a qual não é difícil. Baseia-se nas propriedades da integral e nas dos
polinômios.

Artur Costa Steiner

Em 13 de abr de 2018 00:57, "Mórmon Santos" 
escreveu:

Como é por análise complexa?

Em qui, 12 de abr de 2018 15:22, Artur Steiner <
artur.costa.stei...@gmail.com> escreveu:

> Para n >= 3, só consegui por análise complexa. Há uma prova que me parece
> muito bonita.
>
> Tentei também por frações parciais, mas caí num imbróglio.
>
> Artur Costa Steiner
>
> Em Qui, 12 de abr de 2018 14:42, Claudio Buffara <
> claudio.buff...@gmail.com> escreveu:
>
>> Olá! Alguém encontrou uma solução elementar par este?
>> Eu fiz pra n = 2 e n = 3 mas a generalização me parece muito complicada.
>>
>> []s,
>> Claudio.
>>
>>
>> 2018-04-08 20:42 GMT-03:00 Artur Steiner :
>>
>>> Seja P um polinômio complexo, de grau n >= 2, que tenha n raízes simples
>>> r_1, ... r_n. Mostre que Soma (k = 1, n) 1/P'(r_k) = 0.
>>>
>>> Para quem conhece um pouco de análise complexa, isto é corolário de um
>>> resultado geral. Mas parece que pode ser provado sem análise complexa.
>>>
>>> Artur Costa Steiner
>>>
>>> --
>>> Esta mensagem foi verificada pelo sistema de antivírus e
>>> acredita-se estar livre de perigo.
>>
>>
>>
>> --
>> Esta mensagem foi verificada pelo sistema de antivírus e
>> acredita-se estar livre de perigo.
>
>
> --
> Esta mensagem foi verificada pelo sistema de antivírus e
> acredita-se estar livre de perigo.


-- 
Esta mensagem foi verificada pelo sistema de antivírus e
acredita-se estar livre de perigo.

-- 
Esta mensagem foi verificada pelo sistema de antiv�rus e
 acredita-se estar livre de perigo.



Re: [obm-l] Soma (k = 1, n) 1/P'(r_k) = 0

2018-04-13 Por tôpico Douglas Oliveira de Lima
Entao, sendo x^k=soma (i=1,...,n)(x_i)^k.P(x)/(x-x_i).P'(x_i) , é só
igualar os coeficientes de x^(n-1) e pronto, a identidade se torna ate mais
genérica

Soma (i= 1, n) (x_i)^k/P'(x_i) = 0

Obs: x_i sao raizes.

Abraco

Douglas Oliveira.




Em 8 de abr de 2018 20:50, "Artur Steiner" 
escreveu:

Seja P um polinômio complexo, de grau n >= 2, que tenha n raízes simples
r_1, ... r_n. Mostre que Soma (k = 1, n) 1/P'(r_k) = 0.

Para quem conhece um pouco de análise complexa, isto é corolário de um
resultado geral. Mas parece que pode ser provado sem análise complexa.

Artur Costa Steiner

-- 
Esta mensagem foi verificada pelo sistema de antivírus e
acredita-se estar livre de perigo.

-- 
Esta mensagem foi verificada pelo sistema de antiv�rus e
 acredita-se estar livre de perigo.



Re: [obm-l] Soma (k = 1, n) 1/P'(r_k) = 0

2018-04-12 Por tôpico Mórmon Santos
Como é por análise complexa?

Em qui, 12 de abr de 2018 15:22, Artur Steiner <
artur.costa.stei...@gmail.com> escreveu:

> Para n >= 3, só consegui por análise complexa. Há uma prova que me parece
> muito bonita.
>
> Tentei também por frações parciais, mas caí num imbróglio.
>
> Artur Costa Steiner
>
> Em Qui, 12 de abr de 2018 14:42, Claudio Buffara <
> claudio.buff...@gmail.com> escreveu:
>
>> Olá! Alguém encontrou uma solução elementar par este?
>> Eu fiz pra n = 2 e n = 3 mas a generalização me parece muito complicada.
>>
>> []s,
>> Claudio.
>>
>>
>> 2018-04-08 20:42 GMT-03:00 Artur Steiner :
>>
>>> Seja P um polinômio complexo, de grau n >= 2, que tenha n raízes simples
>>> r_1, ... r_n. Mostre que Soma (k = 1, n) 1/P'(r_k) = 0.
>>>
>>> Para quem conhece um pouco de análise complexa, isto é corolário de um
>>> resultado geral. Mas parece que pode ser provado sem análise complexa.
>>>
>>> Artur Costa Steiner
>>>
>>> --
>>> Esta mensagem foi verificada pelo sistema de antivírus e
>>> acredita-se estar livre de perigo.
>>
>>
>>
>> --
>> Esta mensagem foi verificada pelo sistema de antivírus e
>> acredita-se estar livre de perigo.
>
>
> --
> Esta mensagem foi verificada pelo sistema de antivírus e
> acredita-se estar livre de perigo.

-- 
Esta mensagem foi verificada pelo sistema de antiv�rus e
 acredita-se estar livre de perigo.



Re: [obm-l] Soma (k = 1, n) 1/P'(r_k) = 0

2018-04-12 Por tôpico Artur Steiner
Para n >= 3, só consegui por análise complexa. Há uma prova que me parece
muito bonita.

Tentei também por frações parciais, mas caí num imbróglio.

Artur Costa Steiner

Em Qui, 12 de abr de 2018 14:42, Claudio Buffara 
escreveu:

> Olá! Alguém encontrou uma solução elementar par este?
> Eu fiz pra n = 2 e n = 3 mas a generalização me parece muito complicada.
>
> []s,
> Claudio.
>
>
> 2018-04-08 20:42 GMT-03:00 Artur Steiner :
>
>> Seja P um polinômio complexo, de grau n >= 2, que tenha n raízes simples
>> r_1, ... r_n. Mostre que Soma (k = 1, n) 1/P'(r_k) = 0.
>>
>> Para quem conhece um pouco de análise complexa, isto é corolário de um
>> resultado geral. Mas parece que pode ser provado sem análise complexa.
>>
>> Artur Costa Steiner
>>
>> --
>> Esta mensagem foi verificada pelo sistema de antivírus e
>> acredita-se estar livre de perigo.
>
>
>
> --
> Esta mensagem foi verificada pelo sistema de antivírus e
> acredita-se estar livre de perigo.

-- 
Esta mensagem foi verificada pelo sistema de antiv�rus e
 acredita-se estar livre de perigo.



Re: [obm-l] Soma (k = 1, n) 1/P'(r_k) = 0

2018-04-12 Por tôpico Claudio Buffara
Olá! Alguém encontrou uma solução elementar par este?
Eu fiz pra n = 2 e n = 3 mas a generalização me parece muito complicada.

[]s,
Claudio.


2018-04-08 20:42 GMT-03:00 Artur Steiner :

> Seja P um polinômio complexo, de grau n >= 2, que tenha n raízes simples
> r_1, ... r_n. Mostre que Soma (k = 1, n) 1/P'(r_k) = 0.
>
> Para quem conhece um pouco de análise complexa, isto é corolário de um
> resultado geral. Mas parece que pode ser provado sem análise complexa.
>
> Artur Costa Steiner
>
> --
> Esta mensagem foi verificada pelo sistema de antivírus e
> acredita-se estar livre de perigo.

-- 
Esta mensagem foi verificada pelo sistema de antiv�rus e
 acredita-se estar livre de perigo.



Re: [obm-l] soma de quadrados

2018-03-01 Por tôpico Claudio Buffara
3^2 + 4^2 = 5^2
5^2 + 12^2 = 13^2 ==> 3^2 + 4^2 + 12^2 = 13^2
13^2 + 84^2 = 85^2 ==> 3^2 + 4^2 + 12^2 + 84^2 = 85^2

Em geral, dado a^2 ímpar, você quer x tal que a^2 + x^2 = (x+1)^2 ==> x =
(a^2 -1)/2
a^2 = 85^2 ==> x = (85^2-1)/2 = 3612 ==>  3^2 + 4^2 + 12^2 + 84^2 + 3612^2
= 3613^2

Determinar a sequência que cresce mais devagar é outro problema...



2018-02-28 22:23 GMT-03:00 Bernardo Freitas Paulo da Costa <
bernardo...@gmail.com>:

> 2018-02-28 22:01 GMT-03:00 marcone augusto araújo borges
> :
> > Seja a sequência
> >
> > 3^2 + 4^2 = 5^2
> > 3^2 + 4^2 + 12^2 = 13^2
> > 3^2 + 4^2 + 12^2 + 84^2 = 85^2
> >.
> >.
> >.
> > A soma de n quadrados é um quadrado
> > Existe uma ´´lei de formação´´ ou uma recorrência para determinar
> > uma soma dessas para, digamos, n = 10 ou n = 30 ou n = 100, ...
>
> Vou dar (um) próximo termo.  Não é, necessariamente, o menor, nem o
> melhor, mas ele tem uma "lei de formação" fácil.
>
> 3^2 + 4^2 + 12^2 + 84^2 + 204^2 = 221^2
>
>
> A sequência que eu obtive tem crescimento "exponencial", ou seja, o
> n-ésimo termo é maior do que 2^n.  Seria interessante saber se existe
> uma sequência de crescimento polinomial...
>
> Abraços,
> --
> Bernardo
>
> --
> Esta mensagem foi verificada pelo sistema de antivírus e
>  acredita-se estar livre de perigo.
>
>
> =
> Instru�ões para entrar na lista, sair da lista e usar a lista em
> http://www.mat.puc-rio.br/~obmlistas/obm-l.html
> =
>

-- 
Esta mensagem foi verificada pelo sistema de antiv�rus e
 acredita-se estar livre de perigo.



Re: [obm-l] soma de quadrados

2018-02-28 Por tôpico Bernardo Freitas Paulo da Costa
2018-02-28 22:01 GMT-03:00 marcone augusto araújo borges
:
> Seja a sequência
>
> 3^2 + 4^2 = 5^2
> 3^2 + 4^2 + 12^2 = 13^2
> 3^2 + 4^2 + 12^2 + 84^2 = 85^2
>.
>.
>.
> A soma de n quadrados é um quadrado
> Existe uma ´´lei de formação´´ ou uma recorrência para determinar
> uma soma dessas para, digamos, n = 10 ou n = 30 ou n = 100, ...

Vou dar (um) próximo termo.  Não é, necessariamente, o menor, nem o
melhor, mas ele tem uma "lei de formação" fácil.

3^2 + 4^2 + 12^2 + 84^2 + 204^2 = 221^2


A sequência que eu obtive tem crescimento "exponencial", ou seja, o
n-ésimo termo é maior do que 2^n.  Seria interessante saber se existe
uma sequência de crescimento polinomial...

Abraços,
-- 
Bernardo

-- 
Esta mensagem foi verificada pelo sistema de antiv�rus e
 acredita-se estar livre de perigo.


=
Instru��es para entrar na lista, sair da lista e usar a lista em
http://www.mat.puc-rio.br/~obmlistas/obm-l.html
=


Re: [obm-l] soma de tan^2

2017-09-16 Por tôpico Douglas Oliveira de Lima
Eu resolvi esse problema em 2014 aqui na lista olhe
https://www.mail-archive.com/obm-l@mat.puc-rio.br/msg52281.html

Abraços.

Em 16 de set de 2017 13:23, "Carlos Gomes"  escreveu:

Olá Luis...lembro desse problema ...ele foi publicado na Mathematical
excalibur ha alguns anos https://www.math.ust.hk/excalibur/

A resposta é C(90,2)= 4005, se não me falha a memória...usa relações de
Girard num "polinômio esperto"...vou tenter ver se lembro a solução...se
lembrar ponha aqui!

Abraço, Cgomes.

Em 16 de setembro de 2017 10:48, Luís Lopes 
escreveu:

> Sauda,c~oes,
>
>
> Bom dia.
>
>
> Me mandaram a seguinte questão:
>
>
> (1) Seja S = tan²(1º) + tan²(3º) + tan²(5º) + ... + tan²(89º), calcule o
> valor de S.
>
> Como resolver ? Obrigado.
>
>
> Abs,
>
> Luís
>
>
>
> --
> Esta mensagem foi verificada pelo sistema de antivírus e
> acredita-se estar livre de perigo.
>


-- 
Esta mensagem foi verificada pelo sistema de antivírus e
acredita-se estar livre de perigo.

-- 
Esta mensagem foi verificada pelo sistema de antiv�rus e
 acredita-se estar livre de perigo.



Re: [obm-l] soma de tan^2

2017-09-16 Por tôpico Carlos Gomes
Olá Luis...lembro desse problema ...ele foi publicado na Mathematical
excalibur ha alguns anos https://www.math.ust.hk/excalibur/

A resposta é C(90,2)= 4005, se não me falha a memória...usa relações de
Girard num "polinômio esperto"...vou tenter ver se lembro a solução...se
lembrar ponha aqui!

Abraço, Cgomes.

Em 16 de setembro de 2017 10:48, Luís Lopes 
escreveu:

> Sauda,c~oes,
>
>
> Bom dia.
>
>
> Me mandaram a seguinte questão:
>
>
> (1) Seja S = tan²(1º) + tan²(3º) + tan²(5º) + ... + tan²(89º), calcule o
> valor de S.
>
> Como resolver ? Obrigado.
>
>
> Abs,
>
> Luís
>
>
>
> --
> Esta mensagem foi verificada pelo sistema de antivírus e
> acredita-se estar livre de perigo.
>

-- 
Esta mensagem foi verificada pelo sistema de antiv�rus e
 acredita-se estar livre de perigo.



Re: [obm-l] soma de tan^2

2017-09-16 Por tôpico Israel Meireles Chrisostomo
acho que vc poderia trabalhar na expressão cis(nx) encontrar um polinômio e
usar a relação de girard

Em 16 de setembro de 2017 10:48, Luís Lopes 
escreveu:

> Sauda,c~oes,
>
>
> Bom dia.
>
>
> Me mandaram a seguinte questão:
>
>
> (1) Seja S = tan²(1º) + tan²(3º) + tan²(5º) + ... + tan²(89º), calcule o
> valor de S.
>
> Como resolver ? Obrigado.
>
>
> Abs,
>
> Luís
>
>
>
> --
> Esta mensagem foi verificada pelo sistema de antivírus e
> acredita-se estar livre de perigo.
>



-- 
Israel Meireles Chrisostomo

-- 
Esta mensagem foi verificada pelo sistema de antiv�rus e
 acredita-se estar livre de perigo.



[obm-l] Re: [obm-l] Soma de duas frações irredutíveis

2016-11-25 Por tôpico Pedro José
Boa tarde!

a/b + c/d e (a,b)=1 e (c,d)=1

a/b + c/d = (ad+bc)/bd

Se a/b + c/d é inteiro ==> bd | (ad + bc) ==> b|d e d|b

b| d <=. |b| <= |d|
d | b ==> |d| <= |b|

Então temos que |b| = |d|.

Portanto, creio que deva ser inserida mais uma restrição no problema.

soma de duas frações irredutíveis," ..de denominadores *com módulos *
diferentes"

1/2 + 1/-2 = 0 e 2 <> -2

Saudações,
PJMS


Em 25 de novembro de 2016 10:01, Pedro Chaves 
escreveu:

> Caros Colegas,
>
> Como provar que a soma de duas frações irredutíveis, de denominadores
> diferentes, nunca é um número inteiro?
>
> Abraços!
> Pedro Chaves
>
> --
> Esta mensagem foi verificada pelo sistema de antivírus e
> acredita-se estar livre de perigo.
>

-- 
Esta mensagem foi verificada pelo sistema de antiv�rus e
 acredita-se estar livre de perigo.



Re: [obm-l] soma binomial

2016-08-03 Por tôpico Anderson Torres
Uma ideia que pode funcionar, mas tem que ter alguma base para tentar:
séries formais.
Tente ver se existe uma série formal que descreva f(k), e basta
multiplicá-la por 1/(1-x).



Em 7 de julho de 2016 09:45, Luís Lopes <qed_te...@hotmail.com> escreveu:

> Sauda,c~oes, oi Anderson,
>
>
> > Deve ter alguma forma de passar isso para uma função hipergeométrica
>
> Deve ter. Tentei isso e só complicou.
>
>
> > e ver se de fato tem solução fácil.
>
> Ou melhor, uma solução esperta.
>
> Pelo que sei do problema, deve ter. Vem do
>
> Mathematical Reflections.
>
>
> > Dei uma trapaceada, mas parece que o Wolfram Alpha não reconhece.
>
> Ok. Nem pensei nisso. Mas acho que há programas capazes de
>
> fornecer a forma fechada.
>
>
> > Eu jogo diversos valores e isso tende a 1/3
>
> Verdade. Fica S_n = 1/3 - ??
>
>
> > - e o desejo de usar indução aumenta!
>
> Verdade. A solução apresentada usa indução.
>
> Mas acho nesse caso um pouco bastante artificial
>
> pois o - ?? - acima veio do nada. A indução em si é fácil.
>
>
> Na verdade comecei tentando S_n = \sum_{k=1}^n f(k)
>
> com
>
> f(k) = \frac{ k - 1 } { \binom{2k}{k} }
>
> pois achei que era mais fácil com este f(k) do que com este aqui:
>
> f(k) =  \frac{ 3k + 1 } { ( 2k + 1 ) \binom{2k}{k} } .
>
>
> Neste deu pra calcular F(k) tal que F(k+1) - F(k) = \Delta F(k) = f(k)
>
> e assim S_n = F(n+1) - F(1) = 1 - ?? .
>
>
> Pro f(k) = \frac{ k - 1 } { \binom{2k}{k} } deve ter uma manipulação
>
> binomial esperta pra obter o F(k) que não consigo ver.
>
>
> Abs,
>
> Luís
>
>
>
> --
> *De:* owner-ob...@mat.puc-rio.br <owner-ob...@mat.puc-rio.br> em nome de
> Anderson Torres <torres.anderson...@gmail.com>
> *Enviado:* quinta-feira, 7 de julho de 2016 02:17:43
> *Para:* obm-l@mat.puc-rio.br
> *Assunto:* Re: [obm-l] soma binomial
>
> Deve ter alguma forma de passar isso para uma função hipergeométrica e
> ver se de fato tem solução fácil.
>
> Dei uma trapaceada, mas parece que o Wolfram Alpha não reconhece. Eu
> jogo diversos valores e isso tende a 1/3 - e o desejo de usar indução
> aumenta!
>
> Em 6 de julho de 2016 15:19, Luís Lopes <qed_te...@hotmail.com> escreveu:
> > Sauda,c~oes,
> >
> >
> > Alguém saberia como resolver (sem computador e indução) ?
> >
> > S_n = \sum_{k=1}^n f(k)
> > com
> > f(k) = \frac{ k-1 } { \binom{2k}{k} }.
> >
> > Abs,
> > Luís
> >
> > --
> > Esta mensagem foi verificada pelo sistema de antivírus e
> > acredita-se estar livre de perigo.
>
> --
> Esta mensagem foi verificada pelo sistema de antiv�rus e
>  acredita-se estar livre de perigo.
>
>
> =
> Instru��es para entrar na lista, sair da lista e usar a lista em
> http://www.mat.puc-rio.br/~obmlistas/obm-l.html
> Lista obm-l - Departamento de Matemática - PUC-Rio
> <http://www.mat.puc-rio.br/~obmlistas/obm-l.html>
> www.mat.puc-rio.br
> Lista obm-l. Existem pelo menos dois arquivos da lista obm-l. Um deles
> fica bem aqui, em http://www.mat.puc-rio.br/~obmlistas/obm-l.arquivo.html
>
>
> =
>
> --
> Esta mensagem foi verificada pelo sistema de antivírus e
> acredita-se estar livre de perigo.
>

-- 
Esta mensagem foi verificada pelo sistema de antiv�rus e
 acredita-se estar livre de perigo.



Re: [obm-l] soma binomial

2016-07-07 Por tôpico Luís Lopes
Sauda,c~oes, oi Anderson,


> Deve ter alguma forma de passar isso para uma função hipergeométrica

Deve ter. Tentei isso e só complicou.


> e ver se de fato tem solução fácil.

Ou melhor, uma solução esperta.

Pelo que sei do problema, deve ter. Vem do

Mathematical Reflections.


> Dei uma trapaceada, mas parece que o Wolfram Alpha não reconhece.

Ok. Nem pensei nisso. Mas acho que há programas capazes de

fornecer a forma fechada.


> Eu jogo diversos valores e isso tende a 1/3

Verdade. Fica S_n = 1/3 - ??


> - e o desejo de usar indução aumenta!

Verdade. A solução apresentada usa indução.

Mas acho nesse caso um pouco bastante artificial

pois o - ?? - acima veio do nada. A indução em si é fácil.


Na verdade comecei tentando S_n = \sum_{k=1}^n f(k)

com

f(k) = \frac{ k - 1 } { \binom{2k}{k} }

pois achei que era mais fácil com este f(k) do que com este aqui:

f(k) =  \frac{ 3k + 1 } { ( 2k + 1 ) \binom{2k}{k} } .


Neste deu pra calcular F(k) tal que F(k+1) - F(k) = \Delta F(k) = f(k)

e assim S_n = F(n+1) - F(1) = 1 - ?? .


Pro f(k) = \frac{ k - 1 } { \binom{2k}{k} } deve ter uma manipulação

binomial esperta pra obter o F(k) que não consigo ver.


Abs,

Luís



De: owner-ob...@mat.puc-rio.br <owner-ob...@mat.puc-rio.br> em nome de Anderson 
Torres <torres.anderson...@gmail.com>
Enviado: quinta-feira, 7 de julho de 2016 02:17:43
Para: obm-l@mat.puc-rio.br
Assunto: Re: [obm-l] soma binomial

Deve ter alguma forma de passar isso para uma função hipergeométrica e
ver se de fato tem solução fácil.

Dei uma trapaceada, mas parece que o Wolfram Alpha não reconhece. Eu
jogo diversos valores e isso tende a 1/3 - e o desejo de usar indução
aumenta!

Em 6 de julho de 2016 15:19, Luís Lopes <qed_te...@hotmail.com> escreveu:
> Sauda,c~oes,
>
>
> Alguém saberia como resolver (sem computador e indução) ?
>
> S_n = \sum_{k=1}^n f(k)
> com
> f(k) = \frac{ k-1 } { \binom{2k}{k} }.
>
> Abs,
> Luís
>
> --
> Esta mensagem foi verificada pelo sistema de antivírus e
> acredita-se estar livre de perigo.

--
Esta mensagem foi verificada pelo sistema de antiv?rus e
 acredita-se estar livre de perigo.


=
Instru??es para entrar na lista, sair da lista e usar a lista em
http://www.mat.puc-rio.br/~obmlistas/obm-l.html
Lista obm-l - Departamento de Matemática - 
PUC-Rio<http://www.mat.puc-rio.br/~obmlistas/obm-l.html>
www.mat.puc-rio.br
Lista obm-l. Existem pelo menos dois arquivos da lista obm-l. Um deles fica bem 
aqui, em http://www.mat.puc-rio.br/~obmlistas/obm-l.arquivo.html



=

-- 
Esta mensagem foi verificada pelo sistema de antivírus e
 acredita-se estar livre de perigo.



Re: [obm-l] soma binomial

2016-07-06 Por tôpico Anderson Torres
Deve ter alguma forma de passar isso para uma função hipergeométrica e
ver se de fato tem solução fácil.

Dei uma trapaceada, mas parece que o Wolfram Alpha não reconhece. Eu
jogo diversos valores e isso tende a 1/3 - e o desejo de usar indução
aumenta!

Em 6 de julho de 2016 15:19, Luís Lopes  escreveu:
> Sauda,c~oes,
>
>
> Alguém saberia como resolver (sem computador e indução) ?
>
> S_n = \sum_{k=1}^n f(k)
> com
> f(k) = \frac{ k-1 } { \binom{2k}{k} }.
>
> Abs,
> Luís
>
> --
> Esta mensagem foi verificada pelo sistema de antivírus e
> acredita-se estar livre de perigo.

-- 
Esta mensagem foi verificada pelo sistema de antiv�rus e
 acredita-se estar livre de perigo.


=
Instru��es para entrar na lista, sair da lista e usar a lista em
http://www.mat.puc-rio.br/~obmlistas/obm-l.html
=


[obm-l] Re: [obm-l] Re: [obm-l] Re: [obm-l] Re: [obm-l] Soma dos números naturais

2016-03-03 Por tôpico Leonardo Maia
https://en.wikipedia.org/wiki/1_%2B_2_%2B_3_%2B_4_%2B_%E2%8B%AF

2016-03-03 14:24 GMT-03:00 Sávio Ribas :

> Vi uma palestra sobre isso (entre outras coisas) na última semana. O fato
> é que a Zeta(-1) = -1/12, onde Zeta(s) é a continuação analítica de 1 +
> 1/2^s + 1/3^s + ... para o plano complexo. Essa série só converge se a
> parte real de s é maior que 1, então não faz sentido fazer s = -1 e obter 1
> + 2 + 3 + ... = -1/12. Porém é verdade que alguns experimentos da física
> isso "se torna verdade", no sentido de que algum valor teórico era para dar
> algo que se comporta como 1 + 2 + 3 + ... mas na prática é medido -1/12. O
> exemplo dado na palestra foi de sobreposições de ondas com comprimentos 1,
> 1/2, 1/3, ..., mas a partir daqui não sei mais nada (física, né...).
>
> Em 3 de março de 2016 14:13, Pedro Henrique 
> escreveu:
>
>> Também achei isso mas existem diversos vídeos no YouTube q provam tal
>> afirmação.
>> Em 3 de mar de 2016 2:11 PM, "Alexandre Antunes" <
>> prof.alexandreantu...@gmail.com> escreveu:
>>
>>>
>>> Essa afirmação parece estranha, pois a intuição parece indicar que essa
>>> soma tende para o infinito!
>>> Em 03/03/2016 13:54, "Pedro Henrique"  escreveu:
>>>
 Boa tarde!

 A um bom tempo atrás vi diversas explicações e também aplicações
 práticas na física sobre a soma dos números naturais ser igual a -1/12 mas
 não dei muita importância até que um aluno veio me questionar hj sobre a
 veracidade deste problema, portanto gostaria de saber de vcs se essa
 resposta está realmente correta e se n estiver quais contra-argumentos
 podem ser usados.

 Obrigado!

 --
 Esta mensagem foi verificada pelo sistema de antivírus e
 acredita-se estar livre de perigo.
>>>
>>>
>>> --
>>> Esta mensagem foi verificada pelo sistema de antivírus e
>>> acredita-se estar livre de perigo.
>>
>>
>> --
>> Esta mensagem foi verificada pelo sistema de antivírus e
>> acredita-se estar livre de perigo.
>>
>
>
> --
> Esta mensagem foi verificada pelo sistema de antivírus e
> acredita-se estar livre de perigo.
>

-- 
Esta mensagem foi verificada pelo sistema de antiv�rus e
 acredita-se estar livre de perigo.



[obm-l] Re: [obm-l] Re: [obm-l] Re: [obm-l] Soma dos números naturais

2016-03-03 Por tôpico Sávio Ribas
Vi uma palestra sobre isso (entre outras coisas) na última semana. O fato é
que a Zeta(-1) = -1/12, onde Zeta(s) é a continuação analítica de 1 + 1/2^s
+ 1/3^s + ... para o plano complexo. Essa série só converge se a parte real
de s é maior que 1, então não faz sentido fazer s = -1 e obter 1 + 2 + 3 +
... = -1/12. Porém é verdade que alguns experimentos da física isso "se
torna verdade", no sentido de que algum valor teórico era para dar algo que
se comporta como 1 + 2 + 3 + ... mas na prática é medido -1/12. O exemplo
dado na palestra foi de sobreposições de ondas com comprimentos 1, 1/2,
1/3, ..., mas a partir daqui não sei mais nada (física, né...).

Em 3 de março de 2016 14:13, Pedro Henrique  escreveu:

> Também achei isso mas existem diversos vídeos no YouTube q provam tal
> afirmação.
> Em 3 de mar de 2016 2:11 PM, "Alexandre Antunes" <
> prof.alexandreantu...@gmail.com> escreveu:
>
>>
>> Essa afirmação parece estranha, pois a intuição parece indicar que essa
>> soma tende para o infinito!
>> Em 03/03/2016 13:54, "Pedro Henrique"  escreveu:
>>
>>> Boa tarde!
>>>
>>> A um bom tempo atrás vi diversas explicações e também aplicações
>>> práticas na física sobre a soma dos números naturais ser igual a -1/12 mas
>>> não dei muita importância até que um aluno veio me questionar hj sobre a
>>> veracidade deste problema, portanto gostaria de saber de vcs se essa
>>> resposta está realmente correta e se n estiver quais contra-argumentos
>>> podem ser usados.
>>>
>>> Obrigado!
>>>
>>> --
>>> Esta mensagem foi verificada pelo sistema de antivírus e
>>> acredita-se estar livre de perigo.
>>
>>
>> --
>> Esta mensagem foi verificada pelo sistema de antivírus e
>> acredita-se estar livre de perigo.
>
>
> --
> Esta mensagem foi verificada pelo sistema de antivírus e
> acredita-se estar livre de perigo.
>

-- 
Esta mensagem foi verificada pelo sistema de antiv�rus e
 acredita-se estar livre de perigo.



[obm-l] Re: [obm-l] Re: [obm-l] Soma dos números naturais

2016-03-03 Por tôpico Pedro Henrique
Também achei isso mas existem diversos vídeos no YouTube q provam tal
afirmação.
Em 3 de mar de 2016 2:11 PM, "Alexandre Antunes" <
prof.alexandreantu...@gmail.com> escreveu:

>
> Essa afirmação parece estranha, pois a intuição parece indicar que essa
> soma tende para o infinito!
> Em 03/03/2016 13:54, "Pedro Henrique"  escreveu:
>
>> Boa tarde!
>>
>> A um bom tempo atrás vi diversas explicações e também aplicações práticas
>> na física sobre a soma dos números naturais ser igual a -1/12 mas não dei
>> muita importância até que um aluno veio me questionar hj sobre a veracidade
>> deste problema, portanto gostaria de saber de vcs se essa resposta está
>> realmente correta e se n estiver quais contra-argumentos podem ser usados.
>>
>> Obrigado!
>>
>> --
>> Esta mensagem foi verificada pelo sistema de antivírus e
>> acredita-se estar livre de perigo.
>
>
> --
> Esta mensagem foi verificada pelo sistema de antivírus e
> acredita-se estar livre de perigo.

-- 
Esta mensagem foi verificada pelo sistema de antiv�rus e
 acredita-se estar livre de perigo.



[obm-l] Re: [obm-l] Soma dos números naturais

2016-03-03 Por tôpico Alexandre Antunes
Essa afirmação parece estranha, pois a intuição parece indicar que essa
soma tende para o infinito!
Em 03/03/2016 13:54, "Pedro Henrique"  escreveu:

> Boa tarde!
>
> A um bom tempo atrás vi diversas explicações e também aplicações práticas
> na física sobre a soma dos números naturais ser igual a -1/12 mas não dei
> muita importância até que um aluno veio me questionar hj sobre a veracidade
> deste problema, portanto gostaria de saber de vcs se essa resposta está
> realmente correta e se n estiver quais contra-argumentos podem ser usados.
>
> Obrigado!
>
> --
> Esta mensagem foi verificada pelo sistema de antivírus e
> acredita-se estar livre de perigo.

-- 
Esta mensagem foi verificada pelo sistema de antiv�rus e
 acredita-se estar livre de perigo.



[obm-l] Re: [obm-l] Soma de números compostos

2015-12-13 Por tôpico Carlos Victor
 

Oi Marcone, 

Para n maior do que ou igual a 1, temos: 

i)11+3n = 8+3(n+1) 

ii)11+3n+1 = 9+3(n+1) 

iii) 11+3n+2 = 10+3(n+1) 

Faltando : 12 =8+4 e 13 = 9+4. 

Abraços 

Carlos Victor 

Em 11/12/2015 23:36, marcone augusto araújo borges escreveu: 

> Mostre que todo inteiro n > 11 pode ser escrito como soma de números 
> compostos positivos 
> 
> para n par : n = 11 + 2t-1 = 4 + [2(t + 3)] 
> mas... 
> -- 
> Esta mensagem foi verificada pelo sistema de antivírus e 
> acredita-se estar livre de perigo.
 
-- 
Esta mensagem foi verificada pelo sistema de antiv�rus e
 acredita-se estar livre de perigo.



[obm-l] Re: [obm-l] Soma de números compostos

2015-12-12 Por tôpico Anderson Torres
6K+L, em que L composto percorra as classes de resíduos módulo 6, já
deve servir.

Em 11 de dezembro de 2015 23:36, marcone augusto araújo borges
 escreveu:
> Mostre que todo inteiro n > 11 pode ser escrito como soma de números
> compostos positivos
>
> para n par : n = 11 + 2t-1 = 4 + [2(t + 3)]
> mas...
>
> --
> Esta mensagem foi verificada pelo sistema de antivírus e
> acredita-se estar livre de perigo.

-- 
Esta mensagem foi verificada pelo sistema de antiv�rus e
 acredita-se estar livre de perigo.


=
Instru��es para entrar na lista, sair da lista e usar a lista em
http://www.mat.puc-rio.br/~obmlistas/obm-l.html
=


[obm-l] Re: [obm-l] Soma de números compostos

2015-12-12 Por tôpico Israel Meireles Chrisostomo
Para n ímpar deve seguir que
n=11+2t=9+2(t+1)

Em 11 de dezembro de 2015 23:36, marcone augusto araújo borges <
marconeborge...@hotmail.com> escreveu:

> Mostre que todo inteiro n > 11 pode ser escrito como soma de números
> compostos positivos
>
> para n par : n = 11 + 2t-1 = 4 + [2(t + 3)]
> mas...
>
> --
> Esta mensagem foi verificada pelo sistema de antivírus e
> acredita-se estar livre de perigo.
>

-- 
Esta mensagem foi verificada pelo sistema de antiv�rus e
 acredita-se estar livre de perigo.



[obm-l] RE: [obm-l] Soma de números compostos

2015-12-12 Por tôpico Eduardo Henrique
Cara, acho que todo natural ímpar maior que 11 se escreve como 9+2*n, n natural.

Att., 
Eduardo

From: marconeborge...@hotmail.com
To: obm-l@mat.puc-rio.br
Subject: [obm-l] Soma de números compostos
Date: Sat, 12 Dec 2015 01:36:39 +




Mostre que todo inteiro n > 11 pode ser escrito como soma de númeroscompostos 
positivos
para n par : n = 11 + 2t-1 = 4 + [2(t + 3)]mas...   
  
--

Esta mensagem foi verificada pelo sistema de antivírus e 

 acredita-se estar livre de perigo.


  
-- 
Esta mensagem foi verificada pelo sistema de antivírus e
 acredita-se estar livre de perigo.



[obm-l] RES: [obm-l] RE: [obm-l] Soma de números compostos

2015-12-12 Por tôpico Albert Bouskela - YMAIL
Olá!

Todos os naturais (n) obedecem à seguinte lei de formação:

n = soma [i=0, p] [k(i)x2^i]; k(i)={0, 1}

I.e., todos os naturais podem ser escritos como a soma de potências de 2.
Nesta soma, cada potência de 2 aparece uma, e somente uma, vez. Esta é uma
correspondência biunívoca entre o natural "n" e a respectiva soma das
potências de 2. 

E.g., 23 = (1x2^0) + (1x2^1) + (1x2^2 ) + (0x2^3 ) + (1x2^4)

Sds.,
Albert. 


De: owner-ob...@mat.puc-rio.br [mailto:owner-ob...@mat.puc-rio.br] Em nome
de Eduardo Henrique
Enviada em: sábado, 12 de dezembro de 2015 15:13
Para: obm-l@mat.puc-rio.br
Assunto: [obm-l] RE: [obm-l] Soma de números compostos

Cara, acho que todo natural ímpar maior que 11 se escreve como 9+2*n, n
natural.

Att., 
Eduardo

From: mailto:marconeborge...@hotmail.com
To: mailto:obm-l@mat.puc-rio.br
Subject: [obm-l] Soma de números compostos
Date: Sat, 12 Dec 2015 01:36:39 +
Mostre que todo inteiro n > 11 pode ser escrito como soma de números
compostos positivos

para n par : n = 11 + 2t-1 = 4 + [2(t + 3)]
mas...

-- 
Esta mensagem foi verificada pelo sistema de antivírus e 
acredita-se estar livre de perigo. 

-- 
Esta mensagem foi verificada pelo sistema de antivírus e 
acredita-se estar livre de perigo. 


-- 
Esta mensagem foi verificada pelo sistema de antivírus e
 acredita-se estar livre de perigo.


=
Instruções para entrar na lista, sair da lista e usar a lista em
http://www.mat.puc-rio.br/~obmlistas/obm-l.html
=


Re: [obm-l] Soma

2015-09-29 Por tôpico Rodrigo Renji
Deixo um vídeo com a dedução da fórmula da soma de k=1 até infinito de k
a^k (que dá 1/ (1-a) ).

Daí parece tranquilo obter a que deseja tomando a=e^{-0,08} .

https://www.youtube.com/watch?v=yBRAIuUyM1I=5=PLmT_L9MZaC2mX4fmZwFRuz6RwM8GGNPcS

Em 29 de setembro de 2015 15:38, João Sousa 
escreveu:

> Alguém poderia me passar a fórmula geral para
>
> sum_{k=1}^{\infty} k*exp(-0,08*k)
>
> Abs
> João
>
> --
> Esta mensagem foi verificada pelo sistema de antivírus e
> acredita-se estar livre de perigo.
>

-- 
Esta mensagem foi verificada pelo sistema de antiv�rus e
 acredita-se estar livre de perigo.



Re: [obm-l] soma finita???

2015-06-03 Por tôpico Vitório Batista Lima da Silva
Rapaz ...que sacada ...

Muito obrigado, Ralph



Seja

1S = 1.1+2.2+4.3+8.4+...+2^(n-1).n

Entao, botando um 0 na frente para alinhar do jeito que eu quero:

2S = 0.0+2.1+4.2+8.3+...+2^(n-1).(n-1)+2^n.n

Subtraindo e vendo a PG negativa:

S =  -1 -2 -4 -8... -2^(n-1) + 2^n.n = 2^n.n - 2^n + 1= 2^n.(n-1) + 1

Divida por n, e acabou!



Abraco, Ralph.


-- 
Esta mensagem foi verificada pelo sistema de antivírus e
 acredita-se estar livre de perigo.



Re: [obm-l] Soma de Quadrados

2014-12-19 Por tôpico Ralph Teixeira
Consigo arrumar o problema, mas vai ficar faltando um pedaco no final.

Note que dah para escrever m de forma mais explicita.

m=[n^2-(n-1)^2]+[(n-2)^2-(n-3)^2]+...+[(n-(k-1))^2-(n-k)^2]
onde tem (k+1)/2 pares de termos ali em cima
m=[2n-1]+[2n-5]+[2n-9]+...+[2n-(2k-1)]
m=n(k+1)-[1+5+9+...+(2k-1)]
m=n(k+1)-(2k).(k+1)/2 2 = (2n-k).(k+1)/2

Ou seja, m eh perfeito se for possivel fatora-lo do jeito que estah aa
direita, com nk0 e k impar.

Entao voce TEM que fatorar m=I.J onde I=2n-k eh impar (talvez haja varias
escolhas para I e J -- veremos aa frente); e entao TEM que tomar 2n-k=I e
(k+1)/2=J, isto eh, k=2J+1 e n=(k+I)/2.

Isto dito, SE voce fatorar m=I.J com I impar, voce (quase) sempre PODE
tomar k=2J+1 e n=(k+I)/2! Note que k eh automaticamente positivo e impar!
Ha apenas um problema: precisamos que nk, isto eh, que

(k+I)/2  k
I  k
I  2J+1

Resumindo: se for possivel escrever m=I.J com I impar e I2J+1, entao m eh
interessante. Senao, m NAO eh interessante!

Bom, entao nao perdemos nada se supusermos que I eh o maior fator impar
possivel de m... Ou seja:

i) Escreva m=2^s.I onde I eh impar (s=0, I=1).
ii) Entao m eh interessante se, e somente se, I2^(s+1)+1

Ou seja, os numeros interessantes sao:
a) s=0 implica I=3: 3,5,7,9,11,13,15,17,... (todos os impares exceto 1)
b) s=1 implica I=7: 14,18,22,26,30,34,38,...
c) s=2 implica I=11: 44,52,60,68,76,84,92,...
d) s=3 implica I=19: 136,152,168,184,...
e) s=4 implica I=35...
...

Bom, isso explicita QUEM sao os interessantes, mas ainda fica faltando a
probabilidade... :)

Abraco, Ralph.


2014-12-19 3:43 GMT-02:00 Marcos Martinelli mffmartine...@gmail.com:

 Um número natural m é chamado interessante se existirem n e k naturais
 tais que n  k  0, k é ímpar e ainda:

 m = n^2 - (n - 1)^2 + (n - 2)^2 - ... - (n - k)^2 .

 Seja P_N a probabilidade de escolhermos um número interessante dentre os
 primeiros N naturais.

 Calcular lim (P_N / N) quando N - + infty.



 --
 Esta mensagem foi verificada pelo sistema de antivírus e
 acredita-se estar livre de perigo.

-- 
Esta mensagem foi verificada pelo sistema de antiv�rus e
 acredita-se estar livre de perigo.



Re: [obm-l] Soma de Quadrados

2014-12-19 Por tôpico Ralph Teixeira
Ah, achei um errinho de sinal... :( Deixa eu tentar de novo:

Note que dah para escrever m de forma mais explicita.

m=[n^2-(n-1)^2]+[(n-2)^2-(n-3)^2]+...+[(n-(k-1))^2-(n-k)^2]

onde tem (k+1)/2 pares de termos ali em cima
m=[2n-1]+[2n-5]+[2n-9]+...+[2n-(2k-1)]
m=n(k+1)-[1+5+9+...+(2k-1)]
m=n(k+1)-(2k).(k+1)/2 2 = (2n-k).(k+1)/2

Ou seja, m eh perfeito se for possivel fatora-lo do jeito que estah aa
direita, com nk0 e k impar.

Entao voce TEM que fatorar m=I.J onde I=2n-k eh impar (talvez haja varias
escolhas para I e J -- veremos aa frente); e entao TEM que tomar 2n-k=I e
(k+1)/2=J, isto eh, k=2J-1 e n=(k+I)/2.

Isto dito, SE voce fatorar m=I.J com I impar, voce (quase) sempre PODE
tomar k=2J-1 e n=(k+I)/2! Note que k eh automaticamente positivo e impar!
Ha apenas um problema: precisamos que nk, isto eh, que

(k+I)/2  k
I  k
I  2J-1

Resumindo: se for possivel escrever m=I.J com I impar e I2J-1, entao m eh
interessante. Senao, m NAO eh interessante!

Bom, entao nao perdemos nada se supusermos que I eh o maior fator impar
possivel de m... Ou seja:

i) Escreva m=2^s.I onde I eh impar (s=0, I=1).
ii) Entao m eh interessante se, e somente se, I2^(s+1)-1

Ou seja, os numeros interessantes sao:
a) s=0 implica I=3: 3,5,7,9,11,13,15,17,... (todos os impares exceto 1)
b) s=1 implica I=5: 10,14,18,22,26,30,34,38,...
c) s=2 implica I=9: 36,44,52,60,68,76,84,92,...
d) s=3 implica I=17: 136,152,168,184,...
e) s=4 implica I=33...
...
n) s=n implica I=2^(n+1)+1...
...

Bom, isso explicita QUEM sao os interessantes, mas ainda fica faltando a
probabilidade... :)

Abraco, Ralph.



 2014-12-19 3:43 GMT-02:00 Marcos Martinelli mffmartine...@gmail.com:

 Um número natural m é chamado interessante se existirem n e k naturais
 tais que n  k  0, k é ímpar e ainda:

 m = n^2 - (n - 1)^2 + (n - 2)^2 - ... - (n - k)^2 .

 Seja P_N a probabilidade de escolhermos um número interessante dentre os
 primeiros N naturais.

 Calcular lim (P_N / N) quando N - + infty.



 --
 Esta mensagem foi verificada pelo sistema de antivírus e
 acredita-se estar livre de perigo.




-- 
Esta mensagem foi verificada pelo sistema de antiv�rus e
 acredita-se estar livre de perigo.



Re: [obm-l] Soma de Quadrados

2014-12-19 Por tôpico Marcos Martinelli
Legal. Achei bom o problema.

Principalmente o resultado sobre a densidade dos interessantes.

Em 19 de dezembro de 2014 13:36, Ralph Teixeira ralp...@gmail.com
escreveu:

 Consigo arrumar o problema, mas vai ficar faltando um pedaco no final.

 Note que dah para escrever m de forma mais explicita.

 m=[n^2-(n-1)^2]+[(n-2)^2-(n-3)^2]+...+[(n-(k-1))^2-(n-k)^2]
 onde tem (k+1)/2 pares de termos ali em cima
 m=[2n-1]+[2n-5]+[2n-9]+...+[2n-(2k-1)]
 m=n(k+1)-[1+5+9+...+(2k-1)]
 m=n(k+1)-(2k).(k+1)/2 2 = (2n-k).(k+1)/2

 Ou seja, m eh perfeito se for possivel fatora-lo do jeito que estah aa
 direita, com nk0 e k impar.

 Entao voce TEM que fatorar m=I.J onde I=2n-k eh impar (talvez haja varias
 escolhas para I e J -- veremos aa frente); e entao TEM que tomar 2n-k=I e
 (k+1)/2=J, isto eh, k=2J+1 e n=(k+I)/2.

 Isto dito, SE voce fatorar m=I.J com I impar, voce (quase) sempre PODE
 tomar k=2J+1 e n=(k+I)/2! Note que k eh automaticamente positivo e impar!
 Ha apenas um problema: precisamos que nk, isto eh, que

 (k+I)/2  k
 I  k
 I  2J+1

 Resumindo: se for possivel escrever m=I.J com I impar e I2J+1, entao m eh
 interessante. Senao, m NAO eh interessante!

 Bom, entao nao perdemos nada se supusermos que I eh o maior fator impar
 possivel de m... Ou seja:

 i) Escreva m=2^s.I onde I eh impar (s=0, I=1).
 ii) Entao m eh interessante se, e somente se, I2^(s+1)+1

 Ou seja, os numeros interessantes sao:
 a) s=0 implica I=3: 3,5,7,9,11,13,15,17,... (todos os impares exceto 1)
 b) s=1 implica I=7: 14,18,22,26,30,34,38,...
 c) s=2 implica I=11: 44,52,60,68,76,84,92,...
 d) s=3 implica I=19: 136,152,168,184,...
 e) s=4 implica I=35...
 ...

 Bom, isso explicita QUEM sao os interessantes, mas ainda fica faltando a
 probabilidade... :)

 Abraco, Ralph.


 2014-12-19 3:43 GMT-02:00 Marcos Martinelli mffmartine...@gmail.com:

 Um número natural m é chamado interessante se existirem n e k naturais
 tais que n  k  0, k é ímpar e ainda:

 m = n^2 - (n - 1)^2 + (n - 2)^2 - ... - (n - k)^2 .

 Seja P_N a probabilidade de escolhermos um número interessante dentre os
 primeiros N naturais.

 Calcular lim (P_N / N) quando N - + infty.



 --
 Esta mensagem foi verificada pelo sistema de antivírus e
 acredita-se estar livre de perigo.



 --
 Esta mensagem foi verificada pelo sistema de antivírus e
 acredita-se estar livre de perigo.

-- 
Esta mensagem foi verificada pelo sistema de antiv�rus e
 acredita-se estar livre de perigo.



[obm-l] Re: [obm-l] Re: [obm-l] Re: [obm-l] Soma trigonométrica

2014-06-04 Por tôpico Douglas Oliveira de Lima
Então eu peguei uma dica do Kin Yin Li , quando ele disse pra montar um
polinômio de grau 45 com essas raízes,
Vamos ver, a dica dele se encontra bem aqui
http://www.math.ust.hk/~makyli/2731/2012-2013Sp/2731_LectNt-rev2013.pdf
Eu fiz assim, pensei que
(cosx+isenx)ˆn=cos(nx)+isen(nx)=(cosx)^n+C(n,1)(cosx)^(n-1)(isenx)+C(n,2)(cosx)^(n-2)(isenx)^2+...+(isenx)^n,
dividindo todos os membros por (cosx)^n teríamos
i^n(tgx)^n+i^(n-1)(tgx)^(n-1)+...+1=(cos(nx)+isen(nx))/(cosx)^n, agora para
que tenha raízes ímpares n deve ser 90 , pois cos(90x) tem raízes
ímpares(1,3,5,7,...,179) graus e na igualdade entre as partes reais com n
igual a 90 ficaria
1-C(90,2)(tgx)^2+C(90,4)(tgx)^4-...-(tgx)^90=(cos(nx))/(cosx)^n=0 as raízes
são tg1, tg3, tg5,..., e substitui (tgx)^2 por y ai a equação ficou,
 1-C(90,2)(y)+C(90,4)(y)^2-...-(y)^45=0 cujas raízes são
(tg1)^2,(tg3)^2,(tg5)^2,(tg7)^2,,... cuja soma por girard será C(90,2)=4005.

Desculpe qualquer erro de digitação ou de matemática, acho que é isso daí ,
nas notas do Kyn Yin Li não tem solução, então tive que tentar fazer não
sei se fiou a melhor forma, mas saiu, se puderem me corrigir em alguma
parte que não vi agradeço.

Um abraço!!



Em 3 de junho de 2014 09:16, Bernardo Freitas Paulo da Costa 
bernardo...@gmail.com escreveu:

 2014-06-02 17:48 GMT-03:00 Rogerio Ponce abrlw...@gmail.com:
  Ola' pessoal,
  tem um probleminha que se esqueceram de fazer:

 Esse problema me parece difícil. Eu só consegui fazer usando raízes da
 unidade e polinômios de Chebyshev.

  2014-05-07 8:42 GMT-03:00 Vanderlei Nemitz vanderma...@gmail.com:
 
  Alguém tem alguma ideia? Tentei utilizar a fórmula da tangente do arco
  duplo, mas ficou complicado.
 
  Mostre que tg²(1°) + tg²(3°) + tg²(5°) + ...+ tg²(89°)  é um número
  inteiro.
 

 --
 Bernardo Freitas Paulo da Costa

 --
 Esta mensagem foi verificada pelo sistema de antivírus e
  acredita-se estar livre de perigo.


 =
 Instru�ões para entrar na lista, sair da lista e usar a lista em
 http://www.mat.puc-rio.br/~obmlistas/obm-l.html
 =


-- 
Esta mensagem foi verificada pelo sistema de antiv�rus e
 acredita-se estar livre de perigo.



Re: [obm-l] Re: [obm-l] Re: [obm-l] Soma trigonométrica

2014-06-04 Por tôpico Listeiro 037

Eu andei quebrando a cabeça também e nada conclusivo, mas tem alguns
detalhes que observei:

tg(x) = sen(x)/cos(x) = cos(90-x)/sen(90-x) = cotg(90-x)

Ou seja, tg 1º = 1/tg 89º, não é? Simetria.

Não dá prá fazer tg (1+89) (tg 90º = +oo, talvez algum limite com
L'Hôpital) , ou algo com tg(45-1) = tg(47-3) = tg(49-5)... ? 

tg(45-1) ^ 2 = (tg 45 - tg 1)^2/(1 - tg45 tg 1)^2

O numerador se aproxima do problema.

Em Wed, 4 Jun 2014 21:42:32 -0300
Douglas Oliveira de Lima profdouglaso.del...@gmail.com escreveu:

 Então eu peguei uma dica do Kin Yin Li , quando ele disse pra montar
 um polinômio de grau 45 com essas raízes,
 Vamos ver, a dica dele se encontra bem aqui
 http://www.math.ust.hk/~makyli/2731/2012-2013Sp/2731_LectNt-rev2013.pdf
 Eu fiz assim, pensei que
 (cosx+isenx)ˆn=cos(nx)+isen(nx)=(cosx)^n+C(n,1)(cosx)^(n-1)(isenx)+C(n,2)(cosx)^(n-2)(isenx)^2+...+(isenx)^n,
 dividindo todos os membros por (cosx)^n teríamos
 i^n(tgx)^n+i^(n-1)(tgx)^(n-1)+...+1=(cos(nx)+isen(nx))/(cosx)^n,
 agora para que tenha raízes ímpares n deve ser 90 , pois cos(90x) tem
 raízes ímpares(1,3,5,7,...,179) graus e na igualdade entre as partes
 reais com n igual a 90 ficaria
 1-C(90,2)(tgx)^2+C(90,4)(tgx)^4-...-(tgx)^90=(cos(nx))/(cosx)^n=0 as
 raízes são tg1, tg3, tg5,..., e substitui (tgx)^2 por y ai a equação
 ficou, 1-C(90,2)(y)+C(90,4)(y)^2-...-(y)^45=0 cujas raízes são
 (tg1)^2,(tg3)^2,(tg5)^2,(tg7)^2,,... cuja soma por girard será
 C(90,2)=4005.
 
 Desculpe qualquer erro de digitação ou de matemática, acho que é isso
 daí , nas notas do Kyn Yin Li não tem solução, então tive que tentar
 fazer não sei se fiou a melhor forma, mas saiu, se puderem me
 corrigir em alguma parte que não vi agradeço.
 
 Um abraço!!
 
 
 
 Em 3 de junho de 2014 09:16, Bernardo Freitas Paulo da Costa 
 bernardo...@gmail.com escreveu:
 
  2014-06-02 17:48 GMT-03:00 Rogerio Ponce abrlw...@gmail.com:
   Ola' pessoal,
   tem um probleminha que se esqueceram de fazer:
 
  Esse problema me parece difícil. Eu só consegui fazer usando raízes
  da unidade e polinômios de Chebyshev.
 
   2014-05-07 8:42 GMT-03:00 Vanderlei Nemitz
   vanderma...@gmail.com:
  
   Alguém tem alguma ideia? Tentei utilizar a fórmula da tangente
   do arco duplo, mas ficou complicado.
  
   Mostre que tg²(1°) + tg²(3°) + tg²(5°) + ...+ tg²(89°)  é um
   número inteiro.
  
 
  --
  Bernardo Freitas Paulo da Costa
 
  --
  Esta mensagem foi verificada pelo sistema de antivírus e
   acredita-se estar livre de perigo.
 
 
  =
  Instru�ões para entrar na lista, sair da lista e usar a lista em
  http://www.mat.puc-rio.br/~obmlistas/obm-l.html
  =
 
 



-- 
Encryption works. Properly implemented strong crypto systems are one of
the few things that you can rely on. Unfortunately, endpoint security
is so terrifically weak that NSA can frequently find ways around it. —
Edward Snowden

-- 
Esta mensagem foi verificada pelo sistema de antiv�rus e
 acredita-se estar livre de perigo.


=
Instru��es para entrar na lista, sair da lista e usar a lista em
http://www.mat.puc-rio.br/~obmlistas/obm-l.html
=


[obm-l] Re: [obm-l] Re: [obm-l] Soma trigonométrica

2014-06-03 Por tôpico Bernardo Freitas Paulo da Costa
2014-06-02 17:48 GMT-03:00 Rogerio Ponce abrlw...@gmail.com:
 Ola' pessoal,
 tem um probleminha que se esqueceram de fazer:

Esse problema me parece difícil. Eu só consegui fazer usando raízes da
unidade e polinômios de Chebyshev.

 2014-05-07 8:42 GMT-03:00 Vanderlei Nemitz vanderma...@gmail.com:

 Alguém tem alguma ideia? Tentei utilizar a fórmula da tangente do arco
 duplo, mas ficou complicado.

 Mostre que tg²(1°) + tg²(3°) + tg²(5°) + ...+ tg²(89°)  é um número
 inteiro.


-- 
Bernardo Freitas Paulo da Costa

-- 
Esta mensagem foi verificada pelo sistema de antiv�rus e
 acredita-se estar livre de perigo.


=
Instru��es para entrar na lista, sair da lista e usar a lista em
http://www.mat.puc-rio.br/~obmlistas/obm-l.html
=


[obm-l] Re: [obm-l] Soma trigonométrica

2014-06-02 Por tôpico Rogerio Ponce
Ola' pessoal,
tem um probleminha que se esqueceram de fazer:

a href=http://www.mail-archive.com/obm-l@mat.puc-rio.br/msg52124.html
http://www.mail-archive.com/obm-l@mat.puc-rio.br/msg52124.html
/a

[]'s
Rogerio Ponce


2014-05-07 8:42 GMT-03:00 Vanderlei Nemitz vanderma...@gmail.com:

 Alguém tem alguma ideia? Tentei utilizar a fórmula da tangente do arco
 duplo, mas ficou complicado.


 Mostre que *tg²(1°) + tg²(3°) + tg²(5°) + ...+ tg²(89°)*  é um número
 inteiro.


 Obrigado!

 --
 Esta mensagem foi verificada pelo sistema de antivírus e
 acredita-se estar livre de perigo.

-- 
Esta mensagem foi verificada pelo sistema de antiv�rus e
 acredita-se estar livre de perigo.



[obm-l] Re: [obm-l] Soma trigonométrica

2014-05-07 Por tôpico saulo nilson
=46+d/dxtg(2x+88)(45-somatgxtg(90-x)=46



2014-05-07 8:42 GMT-03:00 Vanderlei Nemitz vanderma...@gmail.com:

 Alguém tem alguma ideia? Tentei utilizar a fórmula da tangente do arco
 duplo, mas ficou complicado.


 Mostre que *tg²(1°) + tg²(3°) + tg²(5°) + ...+ tg²(89°)*  é um número
 inteiro.


 Obrigado!

 --
 Esta mensagem foi verificada pelo sistema de antivírus e
 acredita-se estar livre de perigo.

-- 
Esta mensagem foi verificada pelo sistema de antiv�rus e
 acredita-se estar livre de perigo.



[obm-l] Re: [obm-l] Re: [obm-l] Soma trigonométrica

2014-05-07 Por tôpico Vanderlei Nemitz
O que você fez? Não entendi. Pode detalhar?


Em 7 de maio de 2014 14:49, saulo nilson saulo.nil...@gmail.com escreveu:

 =46+d/dxtg(2x+88)(45-somatgxtg(90-x)=46



 2014-05-07 8:42 GMT-03:00 Vanderlei Nemitz vanderma...@gmail.com:

 Alguém tem alguma ideia? Tentei utilizar a fórmula da tangente do arco
 duplo, mas ficou complicado.


 Mostre que *tg²(1°) + tg²(3°) + tg²(5°) + ...+ tg²(89°)*  é um número
 inteiro.


 Obrigado!

 --
 Esta mensagem foi verificada pelo sistema de antivírus e
 acredita-se estar livre de perigo.



 --
 Esta mensagem foi verificada pelo sistema de antivírus e
 acredita-se estar livre de perigo.

-- 
Esta mensagem foi verificada pelo sistema de antiv�rus e
 acredita-se estar livre de perigo.



RE: [obm-l] soma da Eureka

2013-12-31 Por tôpico Luís



Sauda,c~oes, 
Muito bom, Marcos. Obrigado. 
Pra terminar esta série de msgs, gostaria de tratar do problema 6 na p. 38, 
S(1921) = f(1) + .. + f(1921) para f(k) = 1/(sqr(k) + sqr(k^2 - 1))
Encontrei S(1921) = (sqr(2)/2)(sqr(1922) + sqr(1921) - 1). 
Esta certo? 
Luis 

Date: Mon, 30 Dec 2013 20:34:20 -0200
Subject: Re: [obm-l] soma da Eureka
From: mffmartine...@gmail.com
To: obm-l@mat.puc-rio.br

Na linha seguinte:
* {1/2 . sum{k = 2}^{100} [-1/k + 1/(k - 1)]}
Em segunda-feira, 30 de dezembro de 2013, Marcos Martinelli escreveu:

Uma pequena correção na escrita (quinta linha):
* = 1/2 . f(100) +1/2 . sum{k = 2}^{100} 1/(k^2 - k + 1)


Em segunda-feira, 30 de dezembro de 2013, Marcos Martinelli escreveu:

A gente pode considerar f(k) = (k + 1)/(k^2 + k + 1). 


Podemos mostrar a seguinte relação: 1/(k^4 + k^2 + 1) = 1/2 . [(k + 1)/(k^2 + k 
+ 1) - (k - 1)/(k^2 - k +1)] = 1/2 . [f(k) - f(k - 1) + 1/(k^2 - k +1)] .

Assim, a soma que queremos é tal que: sum{k = 1}^{100} 1/(k^4 + k^2 + 1) = [1/2 
. sum{k = 1}^{100} f(k)] - [1/2 . sum{k = 1}^{100} f(k - 1)] + [1/2 . sum{k = 
1}^{100} 1/(k^2 - k + 1)] = 1/2 . f(100) +1/2 . sum{k = 1}^{100} 1/(k^2 - k + 
1)  1/2 . f(100) + {1/2 . sum{k = 1}^{100} [-1/k + 1/(k - 1)]} = 1/2 . f(100) 
+ 1/2 . (1 - 1/100).



Agora, basta mostrarmos que: 1/2 . f(100) + 1/2 . (1 - 1/100)  1/2 = 
101/10101 + 1 - 1/100  1 = 101/10101  1/100 = 10100  10101 (V). c.q.d






--

Esta mensagem foi verificada pelo sistema de antivírus e 

 acredita-se estar livre de perigo.
  
-- 
Esta mensagem foi verificada pelo sistema de antivírus e
 acredita-se estar livre de perigo.



RE: [obm-l] soma da Eureka

2013-12-30 Por tôpico Luís
Sauda,c~oes, 
Obrigado Marcos. 
No problema 8, f(k) = 1/(k^4 + k^2 + 1). 
Conheço uma forma fechada para g(k) = k/(k^4 + k^2 + 1). 
Como f(k) = g(k) e \sum g(k)  1/2, então  \sum f(k)  1/2. 
Alguém tem outra solução ? 
Luis 

Date: Sun, 29 Dec 2013 22:26:08 -0200
Subject: Re: [obm-l] soma da Eureka
From: mffmartine...@gmail.com
To: obm-l@mat.puc-rio.br

f(x) + f(1 - x) = a^x/(a^x + sqr(a)) + a^(1 - x)/[a^(1 - x) + sqr(a)] = 
a^x/(a^x + sqr(a)) + a/(a + a^x . sqr(a)) = a^x/(a^x + sqr(a)) + sqr(a)/(a^x + 
sqr(a)) = 1.


Em domingo, 29 de dezembro de 2013, Luís escreveu:



Oi, oi Marcos, 
Verdade. O problema 4 tem uma solução parecida: f(x) + f(1/x) = 1. 
E o problema 5 na p. 38 ? f(x) = a^x/(a^x + sqrt(a)). 
Deve ter uma solução usando os argumentos vistos nestas duas últimas soluções. 
Alguma dica? 
Luis 

Date: Sun, 29 Dec 2013 18:20:29 -0200

Subject: Re: [obm-l] soma da Eureka
From: mffmartine...@gmail.com
To: obm-l@mat.puc-rio.br


Para resolver o problema proposto, repare que: f(x) + f(1 - x) = 2/(4^x + 2) + 
2/[4^(1 - x) + 2] = 2/(4^x + 2) + 4^x/(2 + 4^x) = 1.


Em domingo, 29 de dezembro de 2013, Luís escreveu:



Sauda,c~oes, 
Adaptando o problema 3 da p. 37 da Eureka 37, existiria ?? uma forma fechada 
para a soma 
S(n) = a_1 + . + a_n para a_k = \frac{2}{4^k + 2}


Ou também, como fazer o problema proposto ? 
Bom ano para todos. 
Luis 
  
--

Esta mensagem foi verificada pelo sistema de antivírus e 

 acredita-se estar livre de perigo.




--

Esta mensagem foi verificada pelo sistema de antivírus e 

 acredita-se estar livre de perigo.   
--

Esta mensagem foi verificada pelo sistema de antivírus e 

 acredita-se estar livre de perigo.




--

Esta mensagem foi verificada pelo sistema de antivírus e 

 acredita-se estar livre de perigo.   
-- 
Esta mensagem foi verificada pelo sistema de antivírus e
 acredita-se estar livre de perigo.



Re: [obm-l] soma da Eureka

2013-12-30 Por tôpico Marcos Martinelli
A gente pode considerar f(k) = (k + 1)/(k^2 + k + 1).

Podemos mostrar a seguinte relação: 1/(k^4 + k^2 + 1) = 1/2 . [(k + 1)/(k^2
+ k + 1) - (k - 1)/(k^2 - k +1)] = 1/2 . [f(k) - f(k - 1) + 1/(k^2 - k +1)]
.

Assim, a soma que queremos é tal que: sum{k = 1}^{100} 1/(k^4 + k^2 + 1) =
[1/2 . sum{k = 1}^{100} f(k)] - [1/2 . sum{k = 1}^{100} f(k - 1)] + [1/2 .
sum{k = 1}^{100} 1/(k^2 - k + 1)] = 1/2 . f(100) +1/2 . sum{k = 1}^{100}
1/(k^2 - k + 1)  1/2 . f(100) + {1/2 . sum{k = 1}^{100} [-1/k + 1/(k -
1)]} = 1/2 . f(100) + 1/2 . (1 - 1/100).

Agora, basta mostrarmos que: 1/2 . f(100) + 1/2 . (1 - 1/100)  1/2 =
101/10101 + 1 - 1/100  1 = 101/10101  1/100 = 10100  10101 (V). c.q.d

-- 
Esta mensagem foi verificada pelo sistema de antivírus e
 acredita-se estar livre de perigo.



Re: [obm-l] soma da Eureka

2013-12-30 Por tôpico Marcos Martinelli
Uma pequena correção na escrita (quinta linha):

* = 1/2 . f(100) +1/2 . sum{k = 2}^{100} 1/(k^2 - k + 1)

Em segunda-feira, 30 de dezembro de 2013, Marcos Martinelli escreveu:

 A gente pode considerar f(k) = (k + 1)/(k^2 + k + 1).

 Podemos mostrar a seguinte relação: 1/(k^4 + k^2 + 1) = 1/2 . [(k +
 1)/(k^2 + k + 1) - (k - 1)/(k^2 - k +1)] = 1/2 . [f(k) - f(k - 1) + 1/(k^2
 - k +1)] .

 Assim, a soma que queremos é tal que: sum{k = 1}^{100} 1/(k^4 + k^2 + 1) =
 [1/2 . sum{k = 1}^{100} f(k)] - [1/2 . sum{k = 1}^{100} f(k - 1)] + [1/2 .
 sum{k = 1}^{100} 1/(k^2 - k + 1)] = 1/2 . f(100) +1/2 . sum{k = 1}^{100}
 1/(k^2 - k + 1)  1/2 . f(100) + {1/2 . sum{k = 1}^{100} [-1/k + 1/(k -
 1)]} = 1/2 . f(100) + 1/2 . (1 - 1/100).

 Agora, basta mostrarmos que: 1/2 . f(100) + 1/2 . (1 - 1/100)  1/2 =
 101/10101 + 1 - 1/100  1 = 101/10101  1/100 = 10100  10101 (V). c.q.d


-- 
Esta mensagem foi verificada pelo sistema de antivírus e
 acredita-se estar livre de perigo.



Re: [obm-l] soma da Eureka

2013-12-29 Por tôpico Marcos Martinelli
Para resolver o problema proposto, repare que: f(x) + f(1 - x) = 2/(4^x +
2) + 2/[4^(1 - x) + 2] = 2/(4^x + 2) + 4^x/(2 + 4^x) = 1.

Em domingo, 29 de dezembro de 2013, Luís escreveu:

 Sauda,c~oes,

 Adaptando o problema 3 da p. 37 da Eureka 37,
 existiria ?? uma forma fechada para a soma

 S(n) = a_1 + . + a_n para a_k = \frac{2}{4^k + 2}

 Ou também, como fazer o problema proposto ?

 Bom ano para todos.

 Luis


 --
 Esta mensagem foi verificada pelo sistema de antivírus e
 acredita-se estar livre de perigo.


-- 
Esta mensagem foi verificada pelo sistema de antivírus e
 acredita-se estar livre de perigo.



RE: [obm-l] soma da Eureka

2013-12-29 Por tôpico Luís
Oi, oi Marcos, 
Verdade. O problema 4 tem uma solução parecida: f(x) + f(1/x) = 1. 
E o problema 5 na p. 38 ? f(x) = a^x/(a^x + sqrt(a)). Deve ter uma solução 
usando os argumentos vistos nestas duas últimas soluções. 
Alguma dica? 
Luis 

Date: Sun, 29 Dec 2013 18:20:29 -0200
Subject: Re: [obm-l] soma da Eureka
From: mffmartine...@gmail.com
To: obm-l@mat.puc-rio.br

Para resolver o problema proposto, repare que: f(x) + f(1 - x) = 2/(4^x + 2) + 
2/[4^(1 - x) + 2] = 2/(4^x + 2) + 4^x/(2 + 4^x) = 1.


Em domingo, 29 de dezembro de 2013, Luís escreveu:



Sauda,c~oes, 
Adaptando o problema 3 da p. 37 da Eureka 37, existiria ?? uma forma fechada 
para a soma 
S(n) = a_1 + . + a_n para a_k = \frac{2}{4^k + 2}

Ou também, como fazer o problema proposto ? 
Bom ano para todos. 
Luis 
  
--

Esta mensagem foi verificada pelo sistema de antivírus e 

 acredita-se estar livre de perigo.




--

Esta mensagem foi verificada pelo sistema de antivírus e 

 acredita-se estar livre de perigo.   
-- 
Esta mensagem foi verificada pelo sistema de antivírus e
 acredita-se estar livre de perigo.



Re: [obm-l] soma da Eureka

2013-12-29 Por tôpico Marcos Martinelli
f(x) + f(1 - x) = a^x/(a^x + sqr(a)) + a^(1 - x)/[a^(1 - x) + sqr(a)]
= a^x/(a^x
+ sqr(a)) + a/(a + a^x . sqr(a)) = a^x/(a^x + sqr(a)) + sqr(a)/(a^x +
sqr(a)) = 1.

Em domingo, 29 de dezembro de 2013, Luís escreveu:

 Oi, oi Marcos,

 Verdade. O problema 4 tem uma solução parecida:
 f(x) + f(1/x) = 1.

 E o problema 5 na p. 38 ? f(x) = a^x/(a^x + sqrt(a)).
 Deve ter uma solução usando os argumentos vistos
 nestas duas últimas soluções.

 Alguma dica?

 Luis


 --
 Date: Sun, 29 Dec 2013 18:20:29 -0200
 Subject: Re: [obm-l] soma da Eureka
 From: mffmartine...@gmail.com javascript:_e({}, 'cvml',
 'mffmartine...@gmail.com');
 To: obm-l@mat.puc-rio.br javascript:_e({}, 'cvml',
 'obm-l@mat.puc-rio.br');

 Para resolver o problema proposto, repare que: f(x) + f(1 - x) = 2/(4^x +
 2) + 2/[4^(1 - x) + 2] = 2/(4^x + 2) + 4^x/(2 + 4^x) = 1.

 Em domingo, 29 de dezembro de 2013, Luís escreveu:

 Sauda,c~oes,

 Adaptando o problema 3 da p. 37 da Eureka 37,
 existiria ?? uma forma fechada para a soma

 S(n) = a_1 + . + a_n para a_k = \frac{2}{4^k + 2}

 Ou também, como fazer o problema proposto ?

 Bom ano para todos.

 Luis


 --
 Esta mensagem foi verificada pelo sistema de antivírus e
 acredita-se estar livre de perigo.


 --
 Esta mensagem foi verificada pelo sistema de antivírus e
 acredita-se estar livre de perigo.

 --
 Esta mensagem foi verificada pelo sistema de antivírus e
 acredita-se estar livre de perigo.


-- 
Esta mensagem foi verificada pelo sistema de antivírus e
 acredita-se estar livre de perigo.



[obm-l] Re: [obm-l] Re: [obm-l] Re: [obm-l] Soma de radicais irracionais é irracional

2013-09-15 Por tôpico terence thirteen
Então a ideia é provar que o número está num corpo fora de Q? É, parece bem
mais ousada...



Em 9 de setembro de 2013 05:21, Willy George Amaral Petrenko 
wgapetre...@gmail.com escreveu:

 O caso geral é meio complicado. Mas vou dar uma ideia de como se prova que
 √2 + 3√3 é irracional.

 Primeiro introduzimos o conjunto Q[√2], que é o menor corpo que contem
 tanto Q quanto √2. Ele é formado pelos caras da forma a + b√2, onde a,b ∈
 Q. Suponha que √2 + 3√3 ∈ Q[√2]. Então existem a,b tal que √2 + 3√3 = a +
 b√2. Mas então temos que 3√3 ∈ Q[√2] também. Daí temos a,b ∈ Q com 3 = (a
 + b√2)3⇒ 3 = a3. Prossiga de maneira similar a prova tradicional de que 3√
 3 é irracional. Mas nesse caso vc prova √2 +3√3 não pertence a Q[√2]. Mas
 como Q ⊂ Q[√2], temos a resposta.

 Bem, alguns passos são não triviais, mas essa é a ideia.




 2013/9/7 terence thirteen peterdirich...@gmail.com

 Complicadinho...

 Primeiro, dá para supor que a1/m e b1/n estão reduzidos.

 Acho que a forma seria obter um polinômio que tenha esta soma como raiz,
 e provar que nenhum racional pode ser raiz deste polinômio.

 Por exemplo,

 21/2+31/3=x
 81/6+91/6=x

 Assim, podemos de alguma forma supor que x é raiz de um polinômio de grau
 6(acho que alguma coisa relacionada a Álgebra Linear pode provar isto).

 De qualquer forma, calculamos xn, com n de 1 até 6, e tentamos obter
 alguma combinação linear entre as alternativas, para daí obter o polinômio
 de grau 6. MAS como demonstrar que nenhum racional pode ser raiz deste
 polinômio?






 Em 26 de agosto de 2013 19:19, Ennius Lima enn...@bol.com.br escreveu:

 Caros Colegas,

 Sendo a, b, m e n inteiros positivos tais que a1/m e b1/n são
 irracionais, como podemos provar que a soma a1/m + b1/n também é
 irracional?

 Abraços do Ennius!
 __
 Â

 --
 Esta mensagem foi verificada pelo sistema de antivírus e
  acredita-se estar livre de perigo.

 =
 Instruções para entrar na lista, sair da lista e usar a lista em
 http://www.mat.puc-rio.br/~obmlistas/obm-l.html
 =




 --
 /**/
 神が祝福

 Torres

 --
 Esta mensagem foi verificada pelo sistema de antivírus e
 acredita-se estar livre de perigo.



 --
 Esta mensagem foi verificada pelo sistema de antivírus e
 acredita-se estar livre de perigo.




-- 
/**/
神が祝福

Torres

-- 
Esta mensagem foi verificada pelo sistema de antiv�rus e
 acredita-se estar livre de perigo.



[obm-l] Re: [obm-l] Re: [obm-l] Soma de radicais irracionais é irracional

2013-09-09 Por tôpico Willy George Amaral Petrenko
O caso geral é meio complicado. Mas vou dar uma ideia de como se prova que √
2 + 3√3 é irracional.

Primeiro introduzimos o conjunto Q[√2], que é o menor corpo que contem
tanto Q quanto √2. Ele é formado pelos caras da forma a + b√2, onde a,b ∈
Q. Suponha que √2 + 3√3 ∈ Q[√2]. Então existem a,b tal que √2 + 3√3 = a + b√
2. Mas então temos que 3√3 ∈ Q[√2] também. Daí temos a,b ∈ Q com 3 = (a + b√
2)3⇒ 3 = a3. Prossiga de maneira similar a prova tradicional de que 3√3 é
irracional. Mas nesse caso vc prova √2 +3√3 não pertence a Q[√2]. Mas como
Q ⊂ Q[√2], temos a resposta.

Bem, alguns passos são não triviais, mas essa é a ideia.




2013/9/7 terence thirteen peterdirich...@gmail.com

 Complicadinho...

 Primeiro, dá para supor que a1/m e b1/n estão reduzidos.

 Acho que a forma seria obter um polinômio que tenha esta soma como raiz, e
 provar que nenhum racional pode ser raiz deste polinômio.

 Por exemplo,

 21/2+31/3=x
 81/6+91/6=x

 Assim, podemos de alguma forma supor que x é raiz de um polinômio de grau
 6(acho que alguma coisa relacionada a Álgebra Linear pode provar isto).

 De qualquer forma, calculamos xn, com n de 1 até 6, e tentamos obter
 alguma combinação linear entre as alternativas, para daí obter o polinômio
 de grau 6. MAS como demonstrar que nenhum racional pode ser raiz deste
 polinômio?






 Em 26 de agosto de 2013 19:19, Ennius Lima enn...@bol.com.br escreveu:

 Caros Colegas,

 Sendo a, b, m e n inteiros positivos tais que a1/m e b1/n são
 irracionais, como podemos provar que a soma a1/m + b1/n também é
 irracional?

 Abraços do Ennius!
 __
 Â

 --
 Esta mensagem foi verificada pelo sistema de antivírus e
  acredita-se estar livre de perigo.

 =
 Instruções para entrar na lista, sair da lista e usar a lista em
 http://www.mat.puc-rio.br/~obmlistas/obm-l.html
 =




 --
 /**/
 神が祝福

 Torres

 --
 Esta mensagem foi verificada pelo sistema de antivírus e
 acredita-se estar livre de perigo.

-- 
Esta mensagem foi verificada pelo sistema de antiv�rus e
 acredita-se estar livre de perigo.



[obm-l] Re: [obm-l] Soma de radicais irracionais é irracional

2013-09-07 Por tôpico terence thirteen
Complicadinho...

Primeiro, dá para supor que a^(1/m) e b^(1/n) estão reduzidos.

Acho que a forma seria obter um polinômio que tenha esta soma como raiz, e
provar que nenhum racional pode ser raiz deste polinômio.

Por exemplo,

2^(1/2)+3^(1/3)=x
8^(1/6)+9^(1/6)=x

Assim, podemos de alguma forma supor que x é raiz de um polinômio de grau
6(acho que alguma coisa relacionada a Álgebra Linear pode provar isto).

De qualquer forma, calculamos x^n, com n de 1 até 6, e tentamos obter
alguma combinação linear entre as alternativas, para daí obter o polinômio
de grau 6. MAS como demonstrar que nenhum racional pode ser raiz deste
polinômio?






Em 26 de agosto de 2013 19:19, Ennius Lima enn...@bol.com.br escreveu:

 Caros Colegas,

 Sendo a, b, m e n inteiros positivos tais que a^(1/m) e b^(1/n) são
 irracionais, como podemos provar que a soma a^(1/m) + b^(1/n) também é
 irracional?

 Abraços do Ennius!
 __
 Â

 --
 Esta mensagem foi verificada pelo sistema de antivírus e
  acredita-se estar livre de perigo.

 =
 Instruções para entrar na lista, sair da lista e usar a lista em
 http://www.mat.puc-rio.br/~obmlistas/obm-l.html
 =




-- 
/**/
神が祝福

Torres

-- 
Esta mensagem foi verificada pelo sistema de antiv�rus e
 acredita-se estar livre de perigo.



Re: [obm-l] Soma de dois quadrados

2013-07-21 Por tôpico saulo nilson
24^2**2=1152
23^2*2=1058

24^2+23^2=1105
logo 1081 nao pode ser eescrito como soma de 2
a^2+b^2+2ab
(a+b)^2-=1081+2ab
(a+b)^2-2ab
pegando so os 2 uiltimos digitos
so a e b so podem ser par e impar
(2x+1+2y)^2-2(2x+1)2y==provar que nao da um nunca==1
x e y inteiros pertence{0,9}
4x^2+1+4y^2+2(2x+4xy+2y)-4xy-4y==1
4x^2+4y^2++4x+4xy==0
x^2^y^2+x+xy=0
impossivel porque x,y=!0 simultaneamente



2013/7/21 marcone augusto araújo borges marconeborge...@hotmail.com

 Eu testei módulo 8 e já vi que não dá pra escrever 1081 como soma de dois
 quadrados.
 Pensando em não ver apenas assim caso a caso,se não me engano quando um
 número é primo
 ele só é soma de 2 quadrados se for da forma 4k+1

 --
 From: marconeborge...@hotmail.com
 To: obm-l@mat.puc-rio.br
 Subject: [obm-l] Soma de dois quadrados
 Date: Sat, 20 Jul 2013 02:09:39 +


 Gostaria de saber como demonstrar que 1081 não pode ser escrito como soma
 de dois quadrados.
  Eu tentei (a^2 + b^2)(c^2 + d^2) = (ac - bd)^2 + (ad + bc)^2,mas
 1081 = 23.47 e 23 e 47 não são soma de dois quadrados.Lembrei
 inclusive que números da forma 4k+3 não são soma de dois quadrados.

 --
 Esta mensagem foi verificada pelo sistema de antivírus e
 acredita-se estar livre de perigo.

 --
 Esta mensagem foi verificada pelo sistema de antivírus e
 acredita-se estar livre de perigo.


-- 
Esta mensagem foi verificada pelo sistema de antivírus e
 acredita-se estar livre de perigo.



RE: [obm-l] Soma de quadrados

2013-07-18 Por tôpico marcone augusto araújo borges
Achar o menor natural n tal que 2001 é a soma dos quadrados de n 
inteiros(corrigindo)

From: marconeborge...@hotmail.com
To: obm-l@mat.puc-rio.br
Subject: [obm-l] Soma de quadrados
Date: Thu, 18 Jul 2013 19:43:30 +




Achar o menor numero natural n tal que 2001 é a soma de n quadrados
 
 



  


  


  


  


 
 



  


  


  


  

Ideias,por favor. 
--

Esta mensagem foi verificada pelo sistema de antivírus e 

 acredita-se estar livre de perigo.   
-- 
Esta mensagem foi verificada pelo sistema de antivírus e
 acredita-se estar livre de perigo.



Re: [obm-l] Soma de quadrados

2013-07-18 Por tôpico Nilson Carvalho
Provavelmente não é a melhor solução, mas...

44^2+8^2+1^2 = 2001

Vou tentar provar então que 2001 não pode ser escrito como a^2+b^2

Se 2001 = a^2+b^2 = 2001 mod 3 = a^2+b^2 mod 3 = 0 = a^2 + b^2 mod 3. (I)

Como todo quadrado é 0 ou 1 mod 3, a equação  (I) só tem solução se a^2 mod
3 = b^2 mod 3 = 0
a^2 mod 3 = b^2 mod 3 = 0 = a mod 3 = b mod 3 = 0 = a^2 mod 9 = b^2 mod 9
= (a^2+b^2) mod 9 = 0, mas isso não acontece pois 2001 mod 9  0

Att,

Nilson


Em 18 de julho de 2013 17:52, marcone augusto araújo borges 
marconeborge...@hotmail.com escreveu:

 Achar o menor natural n tal que 2001 é a soma dos quadrados de n
 inteiros(corrigindo)

 --
 From: marconeborge...@hotmail.com
 To: obm-l@mat.puc-rio.br
 Subject: [obm-l] Soma de quadrados
 Date: Thu, 18 Jul 2013 19:43:30 +


 Achar o menor numero natural n tal que 2001 é a soma de n quadrados
   Ideias,por favor.

 --
 Esta mensagem foi verificada pelo sistema de antivírus e
 acredita-se estar livre de perigo.

 --
 Esta mensagem foi verificada pelo sistema de antivírus e
 acredita-se estar livre de perigo.


-- 
Esta mensagem foi verificada pelo sistema de antivírus e
 acredita-se estar livre de perigo.



RE: [obm-l] Soma de quadrados

2013-07-18 Por tôpico marcone augusto araújo borges
Poderia  ser tambem 20^2 + 40^2 +1^2Para 2 quadrados eu tinha pensado modulo 
4,modulo 3 ficou melhorValeu,obrigado!

Date: Thu, 18 Jul 2013 18:26:40 -0300
Subject: Re: [obm-l] Soma de quadrados
From: nilson...@gmail.com
To: obm-l@mat.puc-rio.br

Provavelmente não é a melhor solução, mas...
44^2+8^2+1^2 = 2001

Vou tentar provar então que 2001 não pode ser escrito como a^2+b^2
Se 2001 = a^2+b^2 = 2001 mod 3 = a^2+b^2 mod 3 = 0 = a^2 + b^2 mod 3. (I)

Como todo quadrado é 0 ou 1 mod 3, a equação  (I) só tem solução se a^2 mod 3 = 
b^2 mod 3 = 0a^2 mod 3 = b^2 mod 3 = 0 = a mod 3 = b mod 3 = 0 = a^2 mod 9 = 
b^2 mod 9 = (a^2+b^2) mod 9 = 0, mas isso não acontece pois 2001 mod 9  0

Att,
Nilson

Em 18 de julho de 2013 17:52, marcone augusto araújo borges 
marconeborge...@hotmail.com escreveu:




Achar o menor natural n tal que 2001 é a soma dos quadrados de n 
inteiros(corrigindo)

From: marconeborge...@hotmail.com

To: obm-l@mat.puc-rio.br
Subject: [obm-l] Soma de quadrados
Date: Thu, 18 Jul 2013 19:43:30 +




Achar o menor numero natural n tal que 2001 é a soma de n quadrados
 
 



  


  


  


  


 
 



  


  


  


  

Ideias,por favor. 
--

Esta mensagem foi verificada pelo sistema de antivírus e 

 acredita-se estar livre de perigo.   
--

Esta mensagem foi verificada pelo sistema de antivírus e 

 acredita-se estar livre de perigo.





--

Esta mensagem foi verificada pelo sistema de antivírus e 

 acredita-se estar livre de perigo.   
-- 
Esta mensagem foi verificada pelo sistema de antivírus e
 acredita-se estar livre de perigo.



Re: [obm-l] Soma de quadrados

2013-07-18 Por tôpico terence thirteen
Parece meio hard, mas lembrei que

1 - Todo inteiro é soma de 4 quadrados
2 - Muitos inteiros são soma de três quadrados
3 - Alguns tantos são soma de dois quadrados.

Acho que a descrição dos soma-de-três-quadrados é algo como testar módulo
8. Mas, de fato, eu queria tal demonstração em especial.


Em 18 de julho de 2013 21:00, marcone augusto araújo borges 
marconeborge...@hotmail.com escreveu:

 Poderia  ser tambem 20^2 + 40^2 +1^2
 Para 2 quadrados eu tinha pensado modulo 4,modulo 3 ficou melhor
 Valeu,obrigado!

 --
 Date: Thu, 18 Jul 2013 18:26:40 -0300
 Subject: Re: [obm-l] Soma de quadrados
 From: nilson...@gmail.com
 To: obm-l@mat.puc-rio.br


 Provavelmente não é a melhor solução, mas...

 44^2+8^2+1^2 = 2001

 Vou tentar provar então que 2001 não pode ser escrito como a^2+b^2

 Se 2001 = a^2+b^2 = 2001 mod 3 = a^2+b^2 mod 3 = 0 = a^2 + b^2 mod 3. (I)

 Como todo quadrado é 0 ou 1 mod 3, a equação  (I) só tem solução se a^2
 mod 3 = b^2 mod 3 = 0
 a^2 mod 3 = b^2 mod 3 = 0 = a mod 3 = b mod 3 = 0 = a^2 mod 9 = b^2 mod
 9 = (a^2+b^2) mod 9 = 0, mas isso não acontece pois 2001 mod 9  0

 Att,

 Nilson


 Em 18 de julho de 2013 17:52, marcone augusto araújo borges 
 marconeborge...@hotmail.com escreveu:

 Achar o menor natural n tal que 2001 é a soma dos quadrados de n
 inteiros(corrigindo)

 --
 From: marconeborge...@hotmail.com
 To: obm-l@mat.puc-rio.br
 Subject: [obm-l] Soma de quadrados
 Date: Thu, 18 Jul 2013 19:43:30 +


 Achar o menor numero natural n tal que 2001 é a soma de n quadrados
   Ideias,por favor.

 --
 Esta mensagem foi verificada pelo sistema de antivírus e
 acredita-se estar livre de perigo.

 --
 Esta mensagem foi verificada pelo sistema de antivírus e
 acredita-se estar livre de perigo.



 --
 Esta mensagem foi verificada pelo sistema de antivírus e
 acredita-se estar livre de perigo.

 --
 Esta mensagem foi verificada pelo sistema de antivírus e
 acredita-se estar livre de perigo.




-- 
/**/
神が祝福

Torres

-- 
Esta mensagem foi verificada pelo sistema de antiv�rus e
 acredita-se estar livre de perigo.



Re: [obm-l] Soma igual ao produto

2013-06-23 Por tôpico saulo nilson
x+y=xy
x=ky+a
ky+y+a=ky^2+ay
ky^2+y(a-k-1)-a=0

y=(-(a-k-1)+-rq((a--k-1)^2+4ka))/2a
a=1
encontra que nao



2013/5/11 Paulo Argolo pauloarg...@outlook.com

 Caros Colegas,


 Sabemos que 2 + 2 = 2.2  e 1+ 2 + 3 = 1.2.3

 Minha dúvida: Existem outros números reais positivos (dois ou mais,
 distintos ou não) cuja soma seja igual ao produto?


 Abraços do Paulo Argolo

 __
 =
 Instruções para entrar na lista, sair da lista e usar a lista em
 http://www.mat.puc-rio.br/~obmlistas/obm-l.html
 =


-- 
Esta mensagem foi verificada pelo sistema de antivírus e
 acredita-se estar livre de perigo.



RE: [obm-l] Soma igual ao produto

2013-05-14 Por tôpico Paulo Argolo
Caro Ralph,

Convém observar que a afirmação

 Afinal, 1+1+1+...+1+x_1+x_2+...+x_n = 1.1.1.1.1.1.1.x_1.x_2.x_3x_n 
se voce botar o numero certo de 1's ali...

só é válida quando a soma x_1 + x_2 + ... + x_n  for menor do que o produto 
x_1. x_2 . x_3 ... x_n

Bem, uma inevitável perguntinha:

Além dos casos mencionados:  2 + 2 = 2 . 2   e 1 + 2 + 3 = 1. 2. 3 , são 
conhecidos outros exemplos de números naturais, cuja soma é igual ao produto?

Abraços para todos!
Paulo Argolo
_

Date: Sat, 11 May 2013 22:48:00 -0300
Subject: Re: [obm-l] Soma igual ao produto
From: ralp...@gmail.com
To: obm-l@mat.puc-rio.br

Pois eh, {1,2,3} eh bacana porque tem a propriedade e nao eh apelativo que nem 
o meu montao de 1's...

Outro problema eh que, NOS REAIS, voce sempre pode tomar 
x305=(x1+x2+...+x304)/(x1x2x304 - 1). Se o produto x1x304 for maior que 
1, o conjunto {x1,,x305} vai ter a propriedade pedida. Entao o problema nao 
eh tao bacana nos reais, tem respostas demais que nao sao tao especiais...

Entao me parece que a pergunta BACANA eh:

Quais sao as n-uplas (x1,...,xn) (com x1=x2=...=xn) de numeros NATURAIS 
cuja soma eh igual ao produto e que tem NO MAXIMO um numero 1?

(Versao 2, mais facil: SEM nenhum 1?)

Estas eu jah vi em algum lugar -- dah para atacar o problema, e nao tem muitas 
respostas nao. Basicamente, o produto vai ser MUITO maior que a soma, exceto em 
uns poucos casos.

Abraco,
          Ralph


2013/5/11 Bernardo Freitas Paulo da Costa bernardo...@gmail.com
2013/5/11 Ralph Teixeira ralp...@gmail.com:
 Bom, se voce deixar a pergunta assim, a resposta eh sim, montes deltes.

 Afinal, 1+1+1+...+1+x_1+x_2+...+x_n=1.1.1.1.1.1.1.x_1.x_2.x_3x_n se
 voce botar o numero certo de 1's ali...

 Entao a pergunta bacana eh...?

Poxa, eu achei 1 + 2 + 3 = 1 * 2 * 3 tão bacana!
--
Bernardo Freitas Paulo da Costa

=
Instruções para entrar na lista, sair da lista e usar a lista em
http://www.mat.puc-rio.br/~obmlistas/obm-l.html
=   
  
=
Instruções para entrar na lista, sair da lista e usar a lista em
http://www.mat.puc-rio.br/~obmlistas/obm-l.html
=


Re: [obm-l] Soma igual ao produto

2013-05-11 Por tôpico Ralph Teixeira
Bom, se voce deixar a pergunta assim, a resposta eh sim, montes deltes.

Afinal, 1+1+1+...+1+x_1+x_2+...+x_n=1.1.1.1.1.1.1.x_1.x_2.x_3x_n se
voce botar o numero certo de 1's ali...

Entao a pergunta bacana eh...?


2013/5/11 Paulo Argolo pauloarg...@outlook.com

 Caros Colegas,


 Sabemos que 2 + 2 = 2.2  e 1+ 2 + 3 = 1.2.3

 Minha dúvida: Existem outros números reais positivos (dois ou mais,
 distintos ou não) cuja soma seja igual ao produto?


 Abraços do Paulo Argolo

 __
 =
 Instruções para entrar na lista, sair da lista e usar a lista em
 http://www.mat.puc-rio.br/~obmlistas/obm-l.html
 =



Re: [obm-l] Soma igual ao produto

2013-05-11 Por tôpico Bernardo Freitas Paulo da Costa
2013/5/11 Ralph Teixeira ralp...@gmail.com:
 Bom, se voce deixar a pergunta assim, a resposta eh sim, montes deltes.

 Afinal, 1+1+1+...+1+x_1+x_2+...+x_n=1.1.1.1.1.1.1.x_1.x_2.x_3x_n se
 voce botar o numero certo de 1's ali...

 Entao a pergunta bacana eh...?

Poxa, eu achei 1 + 2 + 3 = 1 * 2 * 3 tão bacana!
-- 
Bernardo Freitas Paulo da Costa

=
Instruções para entrar na lista, sair da lista e usar a lista em
http://www.mat.puc-rio.br/~obmlistas/obm-l.html
=


Re: [obm-l] Soma igual ao produto

2013-05-11 Por tôpico Ralph Teixeira
Pois eh, {1,2,3} eh bacana porque tem a propriedade e nao eh apelativo que
nem o meu montao de 1's...

Outro problema eh que, NOS REAIS, voce sempre pode tomar
x305=(x1+x2+...+x304)/(x1x2x304 - 1). Se o produto x1x304 for maior
que 1, o conjunto {x1,,x305} vai ter a propriedade pedida. Entao o
problema nao eh tao bacana nos reais, tem respostas demais que nao sao tao
especiais...

Entao me parece que a pergunta BACANA eh:

Quais sao as n-uplas (x1,...,xn) (com x1=x2=...=xn) de numeros NATURAIS
cuja soma eh igual ao produto e que tem NO MAXIMO um numero 1?

(Versao 2, mais facil: SEM nenhum 1?)

Estas eu jah vi em algum lugar -- dah para atacar o problema, e nao tem
muitas respostas nao. Basicamente, o produto vai ser MUITO maior que a
soma, exceto em uns poucos casos.

Abraco,
  Ralph


2013/5/11 Bernardo Freitas Paulo da Costa bernardo...@gmail.com

 2013/5/11 Ralph Teixeira ralp...@gmail.com:
  Bom, se voce deixar a pergunta assim, a resposta eh sim, montes deltes.
 
  Afinal, 1+1+1+...+1+x_1+x_2+...+x_n=1.1.1.1.1.1.1.x_1.x_2.x_3x_n
 se
  voce botar o numero certo de 1's ali...
 
  Entao a pergunta bacana eh...?

 Poxa, eu achei 1 + 2 + 3 = 1 * 2 * 3 tão bacana!
 --
 Bernardo Freitas Paulo da Costa

 =
 Instruções para entrar na lista, sair da lista e usar a lista em
 http://www.mat.puc-rio.br/~obmlistas/obm-l.html
 =



[obm-l] Re: [obm-l] Re: [obm-l] Soma de funções periódicas

2013-01-19 Por tôpico Jeferson Almir
Ok eu tentei assim.  .

Suponha que $f(0) = g(0) = 0$, que o período de $f$ é $1$ e que o período
de $g$ é um numero $a$ irracional. Seja $b$ o período de $f+g$. Tome um $x$
real qualquer. Voce consegue provar que existe um n inteiro tal que $x +
nb$ está perto de um inteiro e simultaneamente perto de $ma$ para algum $m$
inteiro (tome um $n_0$ tal que x + n_0b dista $epsilon$ de um inteiro,
depois tome o conjuntos dos $n$ inteiros tais que $nb$ dista no maximo
epsilon de um inteiro, dentre estes $n$ voce usa um principio da casa e dos
pombos dividindo a reta em $a/epsilon$ partes para mostrar que há dois
deles que estão na mesma parte (modulo a)...subtraindo você encontra n_1
tal que $n_1b dista no máximo $\epsilon$ (modulo a). Agora voce considera
os multiplos de $n_1b$, da forma $k n_1 b$ até chegar no $k$ conveniente
que te dê $x + k n_1b$ perto (digamos por 2epsilon) modulo $a$.

Ao final você acabará provando que f+g(x) = 0 (usando a continuidade...) e
isso te dará o absurdo.



Em 18 de janeiro de 2013 21:22, Artur Costa Steiner
steinerar...@gmail.comescreveu:

 Obrigado Pedro. Eu me perdi naquela parte da sequência ser densa. Mas, com
 base, na sua idéia, acho que podemos também seguir o seguinte raciocínio.

 No caso de r ser múltiplo racional de p. Conforme mostrado, para todo x,
 g(x + T) = g(x). Isto implica que T = mp seja período de g. Logo, mp é
 múltiplo inteiro de seu período fundamental q. Assim, mp = kq para algum
 inteiro positivo k, do que deduzimos que p/q = k/m é racional. Logo, temos
 uma contradição.

 No caso de r/p e r/q serem irracionais. Aqui eu me perdi, vou analisar
 mais.

 Abraços

 Artur Costa Steiner

 Em 18/01/2013, às 19:26, Pedro Angelo pedro.fon...@gmail.com escreveu:

  Vamos lá..
 
  Imagine que f é periódica de período fundamental p, e g é periódica
 de
  período fundamental q, com p/q irracional, e suponha por absurdo que
  h=f+g é periódica de período r. Então r não pode ser ao mesmo tempo
  múltiplo racional de p e de q. Suponhamos que r não é multiplo inteiro
  de q, ou seja, r/q é irracional.
 
  Digamos que r é múltiplo racional de p, ou seja, existem números
  inteiros n e m não nulos com nr=mp=T. Então f(x+T)=f(x) e h(x+T)=h(x)
  para todo x. Portanto a diferença h-f=g entre eles também satisfaz a
  mesma equação: g(x+T)=g(x) para todo x. Além disso, podemos aumentar
  quantos períodos T quisermos: g(x+kT)=g(x) para todo k inteiro e todo
  x real. É fácil ver onde isso vai chegar: g(x) é constante num
  conjunto denso, e por ser contínua, é constante, o que é absurdo pela
  suposição que você fez.
 
  Digamos então que ambos r/p e r/q são irracionais. Nesse caso, temos
  h(x+kr)=h(x) para todo x. Portanto, f(x)+g(x) = f(x+r) + g(x+r) =
  f(x+2r) + g(x+2r) = etc = h(x). Podemos fixar um x r substituir a
  sequência x+kr por uma sequência a_k contida num período de f, de
  maneira que f(x+kr)=f(a_k), e fazer o mesmo para g, obtendo
  g(x+kr)=g(b_k). Dados quaisquer dois elementos a e b, a dentro do
  período de f onde a_k é denso, e b dentro do período de g onde b_k é
  denso, podemos construir duas subequencias. Uma delas, a'_k, é
  subsequência de a_k convergindo para a, e a outra é uma subsequência
  b'_k de b_k usando somente os índices usados em a'_k. (temos que
  corrigir a'_k para usar somente os índices usados na construção de
  b'_k). Agora, olhamos para o limite quando k vai para infinito de
  f(a'_k)+g(b'_k). Por um lado, essa expressão é igual a h(x) para todo
  k. Por outro, f(a'_k) tende para f(a) e g(b'_k) tende para g(b) (já
  que f e g são contínuas). Portanto, para quaisquer a e b,
  f(a)+g(b)=h(x), ou seja, ambas f e g são constantes! Absurdo!
 
  Foi difícil mas saiu : ) tá um bocado confuso, mas espero que dê pra
  entender. Se alguém tiver uma solução mais simples, eu adoraria ver.
 
  2013/1/18 Artur Costa Steiner steinerar...@gmail.com:
  Eu estou querendo provar isto, mas ainda não cheguei lá não.
 
  Sejam f e g funções de R em R contínuas, periódicas e não
 constantes. Então, f + g é periódica se, e somente se, a relação entre
 os períodos mínimos de f e de g for racional.
 
  A parte se é fácil de mostrar. Para a recíproca, observei que, sendo
 p e q os períodos mínimos de f e de g, então conjunto A = {mp - qn, me e
 n inteiros positivos} , é denso em R. Para todo x, há uma sequênvia em A
 que converge para x, e isso acaba nos mostrando que
 
  lim k -- oo, k inteiro, f(-n_k q) + g(m_k p) = f(x) + g(x)
 
  Mas disto não se conclui que f + g não é periódica.
 
  Abraços
 
  Artur Costa Steiner
 
 =
  Instru�ões para entrar na lista, sair da lista e usar a lista em
  http://www.mat.puc-rio.br/~obmlistas/obm-l.html
 
 =
 
  =
  

[obm-l] Re: [obm-l] Soma de funções periódicas

2013-01-18 Por tôpico Pedro Angelo
Vamos lá..

Imagine que f é periódica de período fundamental p, e g é periódica de
período fundamental q, com p/q irracional, e suponha por absurdo que
h=f+g é periódica de período r. Então r não pode ser ao mesmo tempo
múltiplo racional de p e de q. Suponhamos que r não é multiplo inteiro
de q, ou seja, r/q é irracional.

Digamos que r é múltiplo racional de p, ou seja, existem números
inteiros n e m não nulos com nr=mp=T. Então f(x+T)=f(x) e h(x+T)=h(x)
para todo x. Portanto a diferença h-f=g entre eles também satisfaz a
mesma equação: g(x+T)=g(x) para todo x. Além disso, podemos aumentar
quantos períodos T quisermos: g(x+kT)=g(x) para todo k inteiro e todo
x real. É fácil ver onde isso vai chegar: g(x) é constante num
conjunto denso, e por ser contínua, é constante, o que é absurdo pela
suposição que você fez.

Digamos então que ambos r/p e r/q são irracionais. Nesse caso, temos
h(x+kr)=h(x) para todo x. Portanto, f(x)+g(x) = f(x+r) + g(x+r) =
f(x+2r) + g(x+2r) = etc = h(x). Podemos fixar um x r substituir a
sequência x+kr por uma sequência a_k contida num período de f, de
maneira que f(x+kr)=f(a_k), e fazer o mesmo para g, obtendo
g(x+kr)=g(b_k). Dados quaisquer dois elementos a e b, a dentro do
período de f onde a_k é denso, e b dentro do período de g onde b_k é
denso, podemos construir duas subequencias. Uma delas, a'_k, é
subsequência de a_k convergindo para a, e a outra é uma subsequência
b'_k de b_k usando somente os índices usados em a'_k. (temos que
corrigir a'_k para usar somente os índices usados na construção de
b'_k). Agora, olhamos para o limite quando k vai para infinito de
f(a'_k)+g(b'_k). Por um lado, essa expressão é igual a h(x) para todo
k. Por outro, f(a'_k) tende para f(a) e g(b'_k) tende para g(b) (já
que f e g são contínuas). Portanto, para quaisquer a e b,
f(a)+g(b)=h(x), ou seja, ambas f e g são constantes! Absurdo!

Foi difícil mas saiu : ) tá um bocado confuso, mas espero que dê pra
entender. Se alguém tiver uma solução mais simples, eu adoraria ver.

2013/1/18 Artur Costa Steiner steinerar...@gmail.com:
 Eu estou querendo provar isto, mas ainda não cheguei lá não.

 Sejam f e g funções de R em R contínuas, periódicas e não constantes. Então, 
 f + g é periódica se, e somente se, a relação entre os períodos mínimos de f 
 e de g for racional.

 A parte se é fácil de mostrar. Para a recíproca, observei que, sendo p e q os 
 períodos mínimos de f e de g, então conjunto A = {mp - qn, me e n inteiros 
 positivos} , é denso em R. Para todo x, há uma sequênvia em A que converge 
 para x, e isso acaba nos mostrando que

 lim k -- oo, k inteiro, f(-n_k q) + g(m_k p) = f(x) + g(x)

 Mas disto não se conclui que f + g não é periódica.

 Abraços

 Artur Costa Steiner
 =
 Instru�ões para entrar na lista, sair da lista e usar a lista em
 http://www.mat.puc-rio.br/~obmlistas/obm-l.html
 =

=
Instru��es para entrar na lista, sair da lista e usar a lista em
http://www.mat.puc-rio.br/~obmlistas/obm-l.html
=


  1   2   3   4   >